Con Law MCs

¡Supera tus tareas y exámenes ahora con Quizwiz!

To improve the quality of rental housing within its boundaries, a city proposed an ordinance requiring all new and existing rental housing units to provide at least one full bathroom for each bedroom, plumbing and electrical hookups for a washer and dryer, and a covered parking space. A majority of the owners of existing rental housing in the city opposed the ordinance. They argued that it would dramatically decrease the number of low-income rental housing units because owners would be unable to raise rents enough to recoup the investment required to comply. Without denying these contentions, the city enacted the ordinance. A plaintiff who owns low-income rental housing has sued the city, claiming only that the ordinance is unconstitutional on its face. Which of the following best states the burden of persuasion in this action? A: The city must demonstrate that the ordinance is necessary to serve a compelling state interest, because it adversely affects the fundamental right of rental housing owners to use their property in the manner they deem most economically efficient. B: The city must demonstrate that the ordinance is necessary to serve a compelling state interest, because it will have a substantial and disproportionate negative impact on low-income persons. C: The plaintiff must demonstrate that the ordinance is not substantially related to an important state interest, because it requires some owners of rental housing to invest money that they will not be able to recoup from increased rents. D: The plaintiff must demonstrate that there is no rational relationship between the ordinance and any legitimate state interest, because the ordinance regulates economic activity of a type normally presumed to be within state regulatory authority.

D is correct. The issue presented by this question is to determine which standard of review applies and who has the burden of persuasion. Strict scrutiny applies to regulations involving either suspect classifications or fundamental rights, with the burden of persuasion on the government. Intermediate scrutiny applies when there is a classification based on gender or legitimacy, with the burden of persuasion likely on the government. When the other two standards are inapplicable, then rational basis applies, with the burden of persuasion on the challenger. Here, the lawsuit concerns a city ordinance requiring various accommodations added to rental housing, which does not implicate suspect classifications, classifications based on gender or legitimacy, or fundamental rights. Therefore, the standard of review is rational basis with the burden of persuasion on the owners. A is incorrect. This is the standard for strict scrutiny. Courts use a strict scrutiny standard when a suspect classification or fundamental right is involved. Fundamental rights include the right to travel, privacy, voting, and all First Amendment rights. There is no fundamental right to rental income, so strict scrutiny does not apply here. B is incorrect. This is also the standard for strict scrutiny. Under this standard, a law will only be upheld if it is necessary to achieve a compelling or overriding government purpose. As stated above, strict scrutiny does not apply here. C is incorrect. This is the standard for intermediate scrutiny. Courts use intermediate scrutiny when a classification is based on gender or legitimacy is involved. Under this standard, a law will be upheld if it is substantially related to an important government purpose. Here, neither gender or legitimacy is at issue, and thus intermediate scrutiny does not apply.

A state statute provides that assessments of real property for tax purposes must represent the "actual value" of the property. The County Tax Commission, in making its assessments, has uniformly and consistently determined the "actual value" of real property solely by reference to the price at which the particular property was last sold. In recent years, the market values of real property in the county have been rising at the rate of 15% per year. A homeowner is required to pay real estate taxes on her home in the county that are 200% to 300% higher than those paid by many other owners of similar homes in similar neighborhoods in that county, even though the current market values of their respective homes and the homeowner's home are nearly identical. The reason the taxes on the homeowner's home are higher than those imposed on the other similar homes in similar neighborhoods is that she bought her home much more recently than the other owners and, therefore, it is assessed at a much higher "actual value" than their homes. Persistent efforts by the homeowner to have her assessment reduced or the assessments of the others raised by the County Tax Commission have failed. The homeowner has now filed suit against the County Tax Commission, charging only that the tax assessment on her property is unconstitutional. The strongest constitutional argument to support the homeowner's claim is that the comparative overvaluation of the homeowner's property by the County Tax Commission in making tax assessments over time A: deprives the homeowner of the equal protection of the laws. B: deprives the homeowner of a privilege or immunity of national citizenship. C: constitutes a taking of private property for public use without just compensation. D: constitutes an ex post facto law.

A is correct. A claim that the homeowner's property is being overvalued for tax purposes should rest on equal protection because the homeowner is asserting that she is being treated differently than others in a similar situation. B is incorrect. The homeowner's rights of national citizenship are not implicated in any way here, as the claim involves assessments by the county tax commission of her and her local neighbors' properties. Therefore, the Privileges or Immunities Clause of the Fourteenth Amendment does not apply. C is incorrect. Taxation is almost never a taking of property for public use, and in any event, the homeowner has not been deprived of all economically viable use of her property. D is incorrect. The statute does not retroactively alter the criminal law.

Question A licensed barber works in a particular state. A state barber licensing statute provides that the Barber Licensing Board may revoke a barber license if it finds that the licensee has used his or her business premises for an illegal purpose. The licensed barber was arrested by federal narcotics enforcement agents on a charge of selling cocaine in his barbershop in violation of federal laws. However, the local United States Attorney declined to prosecute and the charges were dropped. Nevertheless, the Barber Licensing Board commenced a proceeding against the barber to revoke his license on the ground that the barber used his business premises for illegal sales of cocaine. At a subsequent hearing before the board, the only pieces of evidence against the barber were affidavits by unnamed informants, who were not present or available for cross-examination. Their affidavits stated that they purchased cocaine from the barber in his barbershop. Based solely on this evidence, the board found that the barber used his business premises for an illegal purpose and ordered his license revoked. In a suit by the barber to have this revocation set aside, his best constitutional argument is A: the barber's inability to cross-examine his accusers denied him a fair hearing and caused him to be deprived of his barber license without due process of law. B: the administrative license revocation proceeding was invalid, because it denied full faith and credit to the dismissal of the criminal charges by the United States Attorney. C: Article II requires a penalty of the kind imposed on the barber to be imposed by a court rather than an administrative agency. D: the existence of federal laws penalizing the illegal sale of cocaine preempts state action relating to drug trafficking of the kind involved in the barber's case.

A is correct. A license is a property right created by state law and cannot be revoked without due process of law. Part of that process may include the ability to cross-examine accusers. Thus, the strongest argument by the barber is that his inability to even know who his accusers were denied him a fair hearing, which deprived him of his barber's license without due process of law. B is incorrect. The decision by the United States Attorney to dismiss the criminal charges is not a law or court judgment that would provide that the state law has not been violated. Thus, the Full Faith and Credit Clause would not apply. C is incorrect. An administrative agency may exercise quasi-judicial powers such as those here. D is incorrect. The federal drug laws are not so pervasive as to preempt the entire subject from state regulation.Moreover, this state regulation is not inconsistent with federal laws, so it would not be preempted.

A federal law provides that all motor vehicle tires discarded in this country must be disposed of in facilities licensed by the federal Environmental Protection Agency. Pursuant to this federal law and all proper federal procedural requirements, that agency has adopted very strict standards for the licensing of such facilities. As a result, the cost of disposing of tires in licensed facilities is substantial. A particular state has a very large fleet of motor vehicles, including trucks used to support state-owned commercial activities and police cars. The state disposes of used tires from both kinds of state motor vehicles in a facility owned and operated by the state. This state facility is unlicensed, but its operation in actual practice meets most of the standards imposed by the federal Environmental Protection Agency on facilities it licenses to dispose of tires. Consistent with United States Supreme Court precedent, may the state continue to dispose of its used tires in this manner? A: No, because a state must comply with valid federal laws that regulate matters affecting interstate commerce. B: No, because some of the tires come from vehicles that are used by the state solely in its commercial activities. C: Yes, because some of the tires come from vehicles that are used by the state in the performance of core state governmental functions such as law enforcement. D: Yes, because the legitimate needs of the federal government are satisfied by the fact that the unlicensed state disposal scheme meets, in actual practice, most of the federal standards for the licensing of such facilities.

A is correct. A state is treated as a person for purposes of federal law and must comply with validly enacted federal laws. This law is validly enacted because tires, which are bought and sold, are part of the stream of commerce. B is incorrect. This answer reaches the correct answer with the wrong reasoning. It is true that the state may not continue to dispose of tires in this manner, but it is not because of the way the tires are used. Rather, it is because the federal law is valid and therefore subjects the state's disposing of the tires to its regulations. C is incorrect. As stated above, the use to which the tires are put is irrelevant. The disposal of tires is not something that is part of the performance of core state governmental functions even if the use of tires is. D is incorrect. It is irrelevant that the state disposal scheme would meet most of the federal standards. The law requires that the disposal facility actually be licensed and that it meet all of the standards.

Congress enacted a statute providing grants of federal funds for the restoration and preservation of courthouses that were built before 1900 and are still in use. The statute contains an inseverable condition requiring that any courthouse restored with the aid of such a grant must be equipped with ramps and other facilities necessary to accommodate physically handicapped people. A law of a particular state requires public buildings in the state to have ramps and other facilities for handicapped people. It exempts from those requirements any building that is more than 70 years old if the State Board of Architects finds that the installation of such facilities would destroy the architectural integrity of the building. The county courthouse in the state was built in 1895 and is still in use. It does not contain ramps or other special facilities for handicapped people. The State Board of Architects has determined that the installation of those facilities would destroy the architectural integrity of the building. The county board applies for a federal grant to restore and preserve that county's courthouse. If the county board restores the courthouse with the aid of a federal restoration and preservation grant, is the board bound to install ramps and other facilities for handicapped people in that building? A: Yes, because Congress may impose reasonable conditions related to the public welfare on grants of federal funds to public bodies when the public bodies are free to accept or reject the grants. B: Yes, because the rights of handicapped and disabled people are fundamental rights that take precedence, as a constitutional matter, over considerations of architectural integrity. C: No, because the Constitution does not authorize the federal government to direct the actions of the states or any of their political subdivisions with respect to matters affecting their own governmental buildings. D: No, because any acceptance of this condition by the county board of supervisors would, as a matter of law, be considered to be under duress.

A is correct. Congress can condition the grant of federal funds to public bodies on compliance with measures related to the public welfare as long as the public body is free to accept or reject the grants. B is incorrect. This is an accurate statement of the law, but it does not answer the call of the question. C is incorrect. There are many ways in which the Constitution would empower Congress to regulate actions concerning state buildings under the Commerce Clause power. D is incorrect. As long as the public body is free to reject the federal funds, the public body is not under duress.

Current national statistics show a dramatic increase in the number of elementary and secondary school students bringing controlled substances (drugs) to school for personal use or distribution to others. In response, Congress enacted a statute requiring each state legislature to enact a state law that makes it a state crime for any person to possess, use, or distribute, within 1,000 feet of any elementary or secondary school, any controlled substance that has previously been transported in interstate commerce and that is not possessed, used, or distributed pursuant to a proper physician's prescription. This federal statute is A: unconstitutional, because Congress has no authority to require a state legislature to enact any specified legislation. B: unconstitutional, because the possession, use, or distribution, in close proximity to a school, of a controlled substance that has previously been transported in interstate commerce does not have a sufficiently close nexus to such commerce to justify its regulation by Congress. C: constitutional, because it contains a jurisdictional provision that will ensure, on a case-by-case basis, that any particular controlled substance subject to the terms of this statute will, in fact, affect interstate commerce. D: constitutional, because Congress possesses broad authority under both the General Welfare Clause and the Commerce Clause to regulate any activities affecting education that also have, in inseverable aggregates, a substantial effect on interstate commerce.

A is correct. Congress does not have the authority to require a state legislature to enact any specified legislation except when it is acting under its spending power. This is not a spending provision, so Congress cannot force states to enact these laws. B is incorrect. The statute only applies to drugs that were transported in interstate commerce. C is incorrect. Even though it may be within Congress's Commerce Clause power to enact the statute it is asking the states to enact, the issue is whether Congress can require the states to enact such legislation. D is incorrect. Congress has no special right to regulate activities affecting education, which is primarily a state function. Moreover, this is not an exercise of Congress's spending power, so it is not supported by the General Welfare Clause.

In order to reduce the federal deficit, Congress enacted a statute imposing a five percent national retail sales tax. The tax was levied upon all retail sales in the United States and applied equally to the sales of all kinds of goods. Is this tax constitutional as applied to retail sales of newspapers? A: Yes, because it is within Congress's power to tax. B: Yes, because the tax is necessary to serve the compelling interest of balancing the federal budget. C: No, because retail sales taxes are within the taxing power of the states. D: No, because the imposition of a tax on the sale of newspapers violates the freedom of the press.

A is correct. Congress has the power to lay and collect taxes. Specifically, the Tax Clause of Article I, Section 8 givesCongress plenary power to raise revenue through taxes, which includes the sale of newspapers. B is incorrect. This answer reaches the correct answer with the wrong reasoning. It is within the power of Congress toraise revenue through taxes, and this power is plenary. Taxes do not need to be necessary to serve compelling interests to be constitutional. C is incorrect. The U.S. Constitution does not reserve retail sales taxes for the states. D is incorrect. Application of the tax to the sale of newspapers does not violate the freedom of the press protected by the First Amendment and trigger strict scrutiny because the tax is generally applicable and in no way targets press operations.

In order to provide funds for a system of new major airports near the ten largest cities in the United States, Congress levies a tax of $25 on each airline ticket issued in the United States. The tax applies to every airline ticket, even those for travel that does not originate in, terminate at, or pass through any of those ten large cities. As applied to the issuance in the United States of an airline ticket for travel between two cities that will not be served by any of the new airports, this tax is A: constitutional, because Congress has broad discretion in choosing the subjects of its taxation and may impose taxes on subjects that have no relation to the purpose for which those tax funds will be expended. B: constitutional, because an exemption for the issuance of tickets for travel between cities that will not be served by the new airports would deny the purchasers of all other tickets the equal protection of the laws. C: unconstitutional, because the burden of the tax outweighs its benefits for passengers whose travel does not originate in, terminate at, or pass through any of the ten largest cities. D: unconstitutional, because the tax adversely affects the fundamental right to travel.

A is correct. Congress's powers to tax and to spend funds are both very broad, and there is no requirement that whatis spent be related in any way to what is taxed. B is incorrect. An exemption for travelers between other cities would not deny any purchasers equal protection of thelaw, since such an exemption would be rationally related to the legitimate governmental purpose of funding the busier airports. C is incorrect. The test is not whether the burden outweighs the benefits for the travelers. In fact, the burdens andbenefits are only relevant if they make the taxing irrational. D is incorrect. The tax does not infringe on the right to travel, since it applies equally to all travel, is a small part of the total cost of plane fare, and there are many other easy ways to travel between the same destinations.

For many years, a city owned and operated a reservoir that supplied the city's residents with water. Based on a finding that private entities were more efficient operators of such facilities than public entities, the city sold the reservoir to a privately owned company. The city council granted the company a non-exclusive franchise to supply water to residents of the city. After it had operated the water system for a year, the company determined that a city resident had fallen three months behind in the payment of his water bill. The company terminated the resident's water service on the basis of a company policy that provides for the summary termination of service to customers who fall more than two months behind in the payment of their bills. The policy does not provide customers an opportunity for a hearing before the termination of service. The resident sued the company in an appropriate court, asking for injunctive relief and damages. The resident claimed only that he had been deprived of property without due process of law in violation of the Fourteenth Amendment. How should the court rule? A: Dismiss the action, because the company is not a state actor for purposes of the Fourteenth Amendment. B: Dismiss the action, because the resident has not proffered payment of any of his arrearage to the company and therefore has no right to continued water service. C: Order the company to give the resident an opportunity for a hearing, because the Fourteenth Amendment applies to private entities performing public functions. D: Order the company to give the resident an opportunity for a hearing, because the Fourteenth Amendment applies to private franchises granted by public entities.

A is correct. For the action of a private entity to satisfy the "state action" requirement, the state must be "significantly involved" in the private entity. The fact that the company supplies a very important good to the public and received a non-exclusive franchise from the city is insufficient to make it a state actor. See Jackson v. Metropolitan Edison Co., 419 U.S. 345 (1974). B is incorrect. Here, the facts do not indicate that the resident was actually in arrears. Also, the purpose of a due process hearing would be to determine whether the resident was in arrears, but the lack of state action means the resident is not entitled to a hearing. C is incorrect. Private entities performing a public function are state actors only if the function is traditionally an exclusive governmental service or one that involves inherently governmental powers such as incarceration. Supplying water does not satisfy either of these requirements. D is incorrect. Receipt of a public franchise is not enough to make a company a state actor. To satisfy the "state action" requirement, the state must be "significantly involved" in the action of the private entity.

In the wake of massive terrorist attacks carried out inside the United States by foreign citizens, Congress declared war on the terrorists' nation of origin. It also passed a statute requiring every alien who is a citizen of the enemy nation to either immediately leave the United States voluntarily or be subject to deportation. An inseverable provision of the new statute provides that the United States Supreme Court will have original and exclusive jurisdiction over any action brought to challenge the validity of the statute. Is the new statute constitutional? A: No, because the statute does not fall within the categories of cases specified in Article III as those over which the Supreme Court shall have original jurisdiction. B: No, because the statute violates the equal protection component of the Fifth Amendment. C: Yes, because among the powers of Congress enumerated in Article I, Section 8, is the power to enact laws governing immigration and naturalization. D: Yes, because Article III specifically provides that the jurisdiction of the Supreme Court shall be subject to such exceptions and regulations as Congress shall make.

A is correct. Granting the U.S. Supreme Court original jurisdiction over actions brought under the statute, which involves deportation of foreign citizens, is not permitted because those issues do not fall within any of the designated categories under Article III establishing the Court's original jurisdiction. B is incorrect. The statute is unconstitutional because it gives improper jurisdiction to the Court over issues not authorized by Article III. However, it is also a misstatement of the law that the statute violates the equal protection of the Fifth Amendment because classifications based on alienage are constitutional as long as they are not arbitrary and unreasonable, and the deportation is rationally related to protecting national security interests. C is incorrect. While Congress does have broad power to regulate matters involving naturalization, this statute nevertheless violates Article III because it grants original jurisdiction to the Court on matters not enumerated in Article III, and Congress may not expand the Court's jurisdiction outside of those categories, as explained above. D is incorrect. This provision of Article III allows for exceptions and regulations regarding the Court's appellate jurisdiction. By contrast, the Court has held that Congress may not expand its original jurisdiction outside of those enumerated categories of Article III.

Congressional committees heard testimony from present and former holders of licenses issued by state vocational licensing boards. According to the testimony, the boards had unfairly manipulated their disciplinary proceedings in order to revoke the licenses of some license holders as a means of protecting favored licensees from competition. In response, Congress enacted a statute prescribing detailed procedural requirements for the disciplinary proceedings of all state vocational licensing boards. The statute requires the state boards to provide licensees with adequate notice and opportunity for an adjudicatory hearing in all disciplinary proceedings. The statute also prescribes membership criteria for state vocational licensing boards, designed to ensure that the boards are likely to be neutral. Which of the following provides the best source of authority for this federal statute? A: Section 5 of the Fourteenth Amendment. B: The general welfare clause of Article I, Section 8. C: The privileges and immunities clause of Article IV, Section 2. D: The takings clause of the Fifth Amendment.

A is correct. Section 5 of the Fourteenth Amendment authorizes this federal statute because it seeks to prevent and/or remedy due process violations against licensees (via favoritism and unfair licensing procedures) and it is congruent with and proportional to that discrimination by the licensing boards. B is incorrect. The General Welfare Clause gives Congress the power to tax and spend for the general welfare, but this power is inapplicable here because the legislation is not a tax or spending measure. C is incorrect. The Privileges and Immunities Clause of Article IV, Section 2 does not apply here because the statute does not address discrimination by licensing boards against the citizens of other states. D is incorrect. The Takings Clause does not apply because this statute does not take anyone's vested interest in property.

The United States had long recognized the ruling faction in a foreign country as that country's government, despite an ongoing civil war. Throughout the civil war, the ruling faction controlled the majority of the country's territory, and the United States afforded diplomatic immunity to the ambassador representing the ruling faction. A newly elected President of the United States decided to recognize a rebel group as the government of the foreign country and notified the ambassador from the ruling faction that she must leave the United States within 10 days. The ambassador filed an action in federal district court for a declaration that the ruling faction was the true government of the foreign country and for an injunction against enforcement of the President's order that she leave the United States. The United States has moved to dismiss the action. If the court dismisses the action, what will be the most likely reason? A: The action involves a nonjusticiable political question. B: The action is not ripe. C: The action is within the original jurisdiction of the U.S. Supreme Court. D: The ambassador does not have standing.

A is correct. The action likely satisfies the political question doctrine and therefore should be dismissed as nonjusticiable. The President's Article II power to receive foreign ambassadors is likely a textually demonstrable commitment by the Constitution of exclusive authority to recognize foreign governments. Moreover, Article II provides no judicially manageable standards by which a court could review the constitutionality of a President's decision on whether to recognize a foreign government. Finally, because the action involves the President's administration of foreign affairs, the prudential elements of the political question doctrine also indicate that the court should dismiss the action as nonjusticiable. B is incorrect. The action is ripe for adjudication even though the ambassador may remain in the United States for 10days. The ambassador has suffered immediate harm because she no longer represents the foreign country in the United States, she has lost her diplomatic immunity, and she is facing expulsion within a very short period of time. Also, the constitutional issues are fit for review without waiting for the ambassador's expulsion. C is incorrect. Although Article III of the Constitution provides that the Supreme Court has original jurisdiction over actions involving ambassadors, federal district courts also may exercise original jurisdiction over actions within the Supreme Court's original jurisdiction. D is incorrect. The ambassador has standing, because she has been injured by the President's decision that her faction is no longer the government of her country, her injury is fairly traceable to this decision, and the injury is likely redressable by a court order invalidating the decision.

A federal statute provides that the United States Supreme Court has authority to review any case filed in a United States Court of Appeals, even though that case has not yet been decided by the court of appeals. The Environmental Protection Agency (EPA), an agency in the executive branch of the federal government, issued an important environmental rule. Although the rule had not yet been enforced against them, companies that would be adversely affected by the rule filed a petition for review of the rule in a court of appeals, seeking a declaration that the rule was invalid solely because it was beyond the statutory authority of the EPA. The companies made no constitutional claim. A statute specifically provides for direct review of EPA rules by a court of appeals without any initial action in a district court. The companies filed a petition for a writ of certiorari in the Supreme Court requesting immediate review of this case by the Supreme Court before the court of appeals has actually decided the case. The EPA acknowledges that the case is important enough to warrant Supreme Court review and that it should be decided promptly, but it asks the Supreme Court to dismiss the petition on jurisdictional grounds. The best constitutional argument in support of the EPA's request is that A: the case is not within the original jurisdiction of the Supreme Court as defined by Article III, and it is not a proper subject of that court's appellate jurisdiction because it has not yet been decided by any lower court. B: the case is appellate in nature, but it is beyond the appellate jurisdiction of the Supreme Court, because Article III states that its jurisdiction extends only to cases arising under the Constitution. C: Article III precludes federal courts from reviewing the validity of any federal agency rule in any proceeding other than an action to enforce the rule. D: Article III provides that all federal cases, except those within the original jurisdiction of the Supreme Court, must be initiated by an action in a federal district court.

A is correct. The case is not within the original jurisdiction of the Court as defined by Article III. It also does not fallwithin the Court's appellate jurisdiction, which would require a lower court decision that was appealed. Therefore, the Supreme Court would not have jurisdiction over this case and the EPA's request should be granted. B is incorrect. As stated above, this is not an issue of appellate jurisdiction because there is no lower court decisionthat has been appealed. Moreover, Article III grants the Court appellate jurisdiction beyond cases arising under the Constitution, but also for cases involving a federal question, and interpretation of a federal rule would be a federal question. C is incorrect. The federal courts can, in fact, review the validity of a federal agency rule in a declaratory judgmentaction as long as the controversy is otherwise justiciable. D is incorrect. Article III does not require that suits be initiated in federal district court.

A federal statute prohibits the sale or resale, in any place in this country, of any product intended for human consumption or ingestion into the human body that contains designated chemicals known to cause cancer, unless the product is clearly labeled as dangerous. The constitutionality of this federal statute may most easily be justified on the basis of the power of Congress to A: regulate commerce among the states. B: enforce the Fourteenth Amendment. C: provide for the general welfare. D: promote science and the useful arts.

A is correct. The power granted to Congress to regulate commerce has been interpreted so broadly that it encompasses almost any economic act that could conceivably have some kind of impact, even secondarily, on the stream of commerce. Because this statute prohibits the sale or resale of certain products without certain safeguards, it clearly falls under this broad power. B is incorrect. The statute seeks to regulate both private and public action, and generally the Fourteenth Amendment only applies to state action. C is incorrect. While Congress can spend for the general welfare, Congress may not directly regulate for the general welfare. D is incorrect. Although Congress is granted the power to promote science and the useful arts, the method by which the Constitution requires that Congress exercise that power is by awarding patents and copyrights, not by regulating the sale of products.

A state law that restricted abortion was challenged in state court as a violation of the due process clause of the Fourteenth Amendment to the U.S. Constitution and as a violation of a similar due process provision of the state constitution. The case made its way to the state's highest court, which ruled that the law violated the due process provisions of both the U.S. and the state constitutions. If petitioned to do so, may the U.S. Supreme Court exercise jurisdiction to review the state court decision? A: No, because the state court's decision in this case rests on adequate and independent state law grounds. B: No, because the U.S. Supreme Court has appellate jurisdiction only over state court decisions that determine the constitutionality of federal laws. C: Yes, because the U.S. Supreme Court has appellate jurisdiction over any ruling of a state's highest court based on an interpretation of federal law. D: Yes, because the U.S. Supreme Court has appellate jurisdiction over decisions that find state laws in violation of the federal Constitution.

A is correct. The state court found the law unconstitutional under both state and federal constitutions, with no indication that the constitutional interpretations were co-extensive. Even if the Court were to reverse as to the federal claim, the outcome of the state court decision would stand given that it found that the law "violated the due process provisions of both the U.S. and the state constitutions" (emphasis added). Therefore, there were adequate and independent grounds supporting the decision, and the Court may not hear the case. B is incorrect. This response correctly states that the Court may not review the decision but inaccurately states the basis for that outcome. The Court's appellate jurisdiction may include state court decisions involving both state and federal claims, but here it does not have jurisdiction because there are independent and adequate state law grounds, as stated above. C is incorrect. The Court may not exercise appellate jurisdiction when the state court decision is based on independent and adequate state-law grounds, even if the decision involved the interpretation of federal law. D is incorrect. As explained above, the Court does not have appellate jurisdiction over a state court decision when an independent and adequate state-law ground exists for the decision.

In order to ensure the high quality of tutors in state public schools, a state law requires public schools to hire only tutors licensed by the state. To obtain a license, a tutor is required to achieve a passing score on a state-administered exam in the tutor's area of specialty. On several occasions, an organization representing tutors licensed in the state has successfully lobbied against proposed legislation that would have eased the licensing requirement. An out-of-state tutoring company would like to conduct business in the state, but very few of its tutors are licensed by the state. The company has sued to challenge the law. Is the company likely to prevail? A: No, because the law is rationally related to a legitimate state interest. B: No, because the law is substantially related to an important state interest. C: Yes, because the law serves the special interests of licensed tutors rather than a legitimate government interest. D: Yes, because the law violates the privileges and immunities clause of Article IV.

A is correct. The state licensing requirement should receive rational basis scrutiny because it neither burdens a fundamental individual right nor creates a constitutionally suspect classification. The licensing requirement is a rational means of protecting the state's legitimate interest in ensuring the competency of tutors. Therefore, the state law satisfies rational basis review and is constitutional. B is incorrect. This answer describes the more demanding standard of review, intermediate judicial scrutiny. The licensing requirement for tutors neither burdens a specially protected individual right nor creates a constitutionally sensitive classification and should receive rational basis review. C is incorrect. Although the licensing requirement serves the special interests of licensed tutors, it also serves the state's legitimate interest in ensuring the competency of tutors, which is all that is required to sustain the statute. D is incorrect. The Privileges and Immunities Clause of Article IV, which prohibits certain forms of discrimination by states against the citizens of other states, is not implicated by these facts. Although the challenge has been brought by an out-of-state company, the state licensing requirement applies equally to the state's citizens and to the citizens of other states.

A state law requires that anyone convicted for the second time of a sex offense serve a specified minimum prison sentence and be sterilized. Statistics show that 50% of those convicted of sex offenses in the state over the past five years are members of racial minority groups, whereas the members of those groups account for only 15% of the state's general population. A significant majority of convicted sex offenders are male. Which of the following would be the strongest argument in challenging the constitutionality of the law under the equal protection clause of the Fourteenth Amendment? A: The requirement of sterilization deprives a select group of persons of a fundamental right, and it is not necessary to serve a compelling governmental interest. B: The requirement of sterilization has a disproportionate impact on men, and it is not substantially related to an important governmental interest. C: The requirement of sterilization has a disproportionate impact on racial minorities, and it is not necessary to serve a compelling governmental interest. D: The requirement of sterilization is not rationally related to a legitimate governmental interest.

A is correct. The state requirement of sterilization denies affected individuals the right to procreate, which the USSupreme Court holds to be a fundamental right. See Skinner v. Oklahoma, 316 U.S. 535 (1942). A state law that burdens a fundamental right must satisfy strict judicial scrutiny, meaning it must be necessary to serve a compelling governmental interest. Therefore, an argument that the law does not pass strict scrutiny would be the strongest argument. B is incorrect. This answer choice states the elements of intermediate judicial scrutiny, which is not the proper standard. Although laws that discriminate on the basis of sex receive intermediate scrutiny, a sex-neutral law like the sterilization requirement here constitutes sex discrimination only if the legislature intended that the requirement benefit or harm individuals because of their sex. There is no indication of such an intention from these facts. The disparate impact of the sterilization requirement on men is, by itself, insufficient to demonstrate sex discrimination. C is incorrect. The state law receives strict judicial scrutiny because it burdens the fundamental right of procreation, not because it is racially discriminatory. A race-neutral law like the sterilization requirement triggers strict scrutiny only if the legislature intended that the requirement benefit or harm individuals due to their race. There is no indication of such an intention from these facts. The disparate impact of the sterilization requirement on members of a racial minority group is, by itself, insufficient to demonstrate race discrimination. D is incorrect. This answer choice states the elements of rational basis scrutiny, which is not applicable in this case.The state sterilization requirement receives strict judicial scrutiny because it burdens the fundamental right of procreation.

A state law provides that only U.S. citizens may serve as jurors in the state courts of that state. A woman who is a lawful resident alien and who has resided in the state for many years was summoned for jury duty in a state court. The woman's name was selected from a list of potential jurors that was compiled from a comprehensive list of local residents. She was disqualified from service solely because she is not a U.S. citizen. The woman has filed an action for a declaratory judgment that the state law is unconstitutional. Who should prevail in this action? A: The state, because a state may limit to U.S. citizens functions that are an integral part of the process of self-government. B: The state, because jury service is a privilege, not a right, and therefore it is not a liberty interest protected by the due process clause of the Fourteenth Amendment. C: The woman, because the Constitution gives Congress plenary power to make classifications with respect to aliens. D: The woman, because the state has not articulated a legitimate reason for prohibiting resident aliens from serving as jurors in the state's courts.

A is correct. The state should prevail because the law excluding aliens from jury service is rationally related to the state's legitimate interest in ensuring that only citizens perform functions that are central to self-government. Although strict scrutiny generally applies to state laws that discriminate against aliens, rational basis scrutiny is appropriate when alienage classifications restrict the right to participate in functions that are central to self-government, such as voting, running for office, or serving on a jury. B is incorrect. Application of the Due Process Clause no longer turns on whether the individual interest involved is aright or a privilege. In any event, the woman's constitutional challenge to the state law should be based on the Equal Protection Clause rather than on the Due Process Clause. While the state should prevail, it is because the law excluding aliens from jury service is rationally related to the state's legitimate interest in ensuring that only citizens perform functions that are central to self-government. C is incorrect. Although the Constitution gives Congress plenary power to control immigration, states may exercise their police powers to regulate the conduct of aliens within their borders unless the regulation is preempted by federal law or otherwise violates the Constitution. While strict scrutiny generally applies to state laws that discriminate against aliens, rational basis scrutiny is appropriate when alienage classifications restrict the right to participate in functions that are central to self-government, as explained above. D is incorrect. The state's interest in ensuring that only citizens perform functions central to self-government is considered legitimate under the rational basis test, as stated above.

A particular state has a state employee grievance system that requires any state employee who wishes to file a grievance against the state to submit that grievance for final resolution to a panel of three arbitrators chosen by the parties from a statewide board of 13 arbitrators. In any given case, the grievant and the state alternate in exercising the right of each party to eliminate five members of the board, leaving a panel of three members to decide their case. At the present time, the full board is composed of seven male arbitrators and six female arbitrators. A female state employee filed a sexual harassment grievance against her male supervisor and the state. The state's attorney exercised all of her five strikes to eliminate five of the female arbitrators. At the time she did so, the state's attorney stated that she struck the five female arbitrators solely because she believed women, as a group, would necessarily be biased in favor of another woman who was claiming sexual harassment. Counsel for the state employee eliminated four males and one female arbitrator, all solely on the grounds of specific bias or conflicts of interest. As a result, the panel was all male. When the panel ruled against the state employee on the merits of her case, she filed an action in an appropriate state court, challenging the panel selection process as a gender-based denial of equal protection of the laws. In this case, the court should hold that the panel selection process is A: unconstitutional, because the gender classification used by the state's attorney in this case does not satisfy the requirements of intermediate scrutiny. B: unconstitutional, because the gender classification used by the state's attorney in this case denies the grievant the right to a jury made up of her peers. C: constitutional, because the gender classification used by the state's attorney in this case satisfies the requirements of the strict scrutiny test. D: constitutional, because the gender classification used by the state's attorney in this case satisfies the requirements of the rational basis test.

A is correct. The state's attorney in this case intentionally excluded arbitrators based on gender. The Supreme Court has held that peremptory challenges based solely on gender are unconstitutional as a violation of the Equal Protection Clause because they reinforce negative stereotypes against women without furthering the governmental interest of a fair trial. See J.E.B. v. Alabama, 511 U.S. 127 (1994). Similarly, the state's attorney in this question is making a gender-based classification, so the state would need to show that the classification is substantially related to an important government objective. Because the state's attorney's argument was that women would simply be biased toward a female claimant, the attorney failed to demonstrate how the exclusion of female arbitrators would be substantially related to the need for a fair arbitration. B is incorrect. The constitutional right to a jury of one's peers only applies to formal jury trials, not arbitrations. C is incorrect. Strict scrutiny does not apply. Since this is a gender-based classification, it is subject to intermediate scrutiny. D is incorrect. Rational basis is the improper test to apply to a gender-based classification. Intermediate scrutiny applies, so the government would need to show that the classification is substantially related to an important government objective.

A federal statute requires any individual or entity owning more than 100 cars to ensure that at least 10 percent of those cars are electric-powered. A city has sued the federal official responsible for enforcing this statute in federal district court, seeking an injunction prohibiting enforcement of the statute on the ground that it is unconstitutional. Should the court grant the injunction? A: No, because the statute is valid under the commerce clause and does not violate the Tenth Amendment. B: No, because the federal government has sovereign immunity and cannot be sued without its explicit consent. C: Yes, because the statute violates the reserved rights of the states under the Tenth Amendment. D: Yes, because as applied to state and local governments, the statute exceeds Congress's power under the commerce clause.

A is correct. The statute falls within the scope of congressional power to regulate commercial activity that, in theaggregate, has a substantial effect on interstate commerce. B is incorrect. Although the federal government does enjoy sovereign immunity from being sued as a party, thisimmunity applies only to suits seeking compensatory monetary relief and not to suits seeking only prospective injunctive relief. The suit here seeks only prospective injunctive relief. C is incorrect. The statute does not violate the Tenth Amendment, which protects local governments from beingcommandeered by the federal government. This statute both regulates commercial activity having a substantial effect on interstate commerce and applies consistently to all entities engaging in the same type of conduct. D is incorrect. On the contrary, this statute falls squarely within congressional commerce powers to regulate activities having a substantial effect on interstate commerce, as explained above.

An ordinance of a particular city requires that its mayor have continuously been a resident of the city for at least five years at the time he or she takes office. A candidate, who is thinking about running for mayor in an election that will take place next year, will have been a resident of the city for only four and one-half years at the time the mayor elected then takes office. Before he decides whether to run for the position of mayor, the candidate wants to know whether he could lawfully assume that position if he were elected. As a result, the candidate files suit in the local federal district court for a declaratory judgment that the city's five-year-residence requirement is unconstitutional and that he is entitled to a place on his political party's primary election ballot for mayor. He names the chairman of his political party as the sole defendant but does not join any election official. The chairman responds by joining the candidate in requesting the court to declare the city's residence requirement invalid. In this case, the court should A: refuse to determine the merits of this suit, because there is no case or controversy. B: refuse to issue such a declaratory judgment, because an issue of this kind involving only a local election does not present a substantial federal constitutional question. C: issue the declaratory judgment, because a residency requirement of this type is a denial of the equal protection of the laws. D: issue the declaratory judgment, because the candidate will have substantially complied with the residency requirement.

A is correct. There is no case or controversy here because the candidate lacks standing. One reason he lacks standing is that the issue is not yet ripe for review. The issue is not ripe because the candidate will not suffer an injury until he is barred from running for office or until he has won the election. At that point, an injury will either have occurred or will be so certain to occur as to confer standing. The second reason the candidate lacks standing is that the candidate has not joined the correct party to provide him the relief that will redress the injury. The chairman of his local political party would be the correct party to sue if the political party refuses to put him on the ballot due to the residency requirement. However, the candidate seems to be challenging the qualifications with the assumption that he will not be allowed to take office. The correct defendant would be whichever official enforces those qualifications and determines whether elected officials take office, presumably election officials. Finally, the mere agreement by the parties that the court should consider the issue is irrelevant; standing cannot be created by agreement. B is incorrect. The qualifications of an elected official, even for a local election, do present a substantial federal constitutional question. C is incorrect. The court may not reach the merits of the case if the candidate lacks standing. Moreover, it is not clear that this residency requirement would deny candidates equal protection of the law. D is incorrect. The federal courts may not modify the state law by finding substantial compliance enough if the state law is otherwise constitutional.

Under the authority of a federal voting rights statute, some states drew congressional districts in a manner calculated to increase the likelihood that members of historically disadvantaged minority racial groups would be elected. The U.S. Supreme Court declared these districts to be unconstitutional, as improper racial gerrymanders. In response to this ruling, Congress passed a new statute that explicitly denies the Supreme Court appellate jurisdiction over all future cases challenging the constitutionality of action taken under the authority of the federal voting rights statute. Which of the following is the most persuasive argument for the constitutionality of the new statute restricting the Supreme Court's appellate jurisdiction? A: Article III of the Constitution explicitly states that the Supreme Court's appellate jurisdiction is subject to such exceptions and regulations as Congress shall make. B: The constitutional principle of separation of powers authorizes Congress to pass statutes calculated to reduce the effects of Supreme Court decisions that interfere with the exercise of powers that have been delegated to the legislative branch. C: The establishment and apportionment of congressional districts directly affect interstate commerce, and the Constitution authorizes Congress to use its plenary authority over such commerce for any purpose it believes will promote the general welfare. D: The Fifteenth Amendment authorizes Congress to enforce the amendment's voting rights provisions by appropriate legislation, and Congress could reasonably determine that this restriction on the Supreme Court's appellate jurisdiction is an appropriate means to that end.

A is correct. This is the most persuasive argument because Congress has the authority to regulate and limit theCourt's appellate jurisdiction pursuant to Article III and Ex parte McCardle. B is incorrect. The separation of powers prevents the branches of government from overstepping and interfering withone another. It prohibits the legislature from interfering with the courts' final judgments, it does not authorize Congress to limit the Court's appellate jurisdiction. C is incorrect. The Commerce Clause gives Congress the power to regulate commerce among the several states. Theapportionment of congressional districts does not directly affect interstate commerce. Therefore, this Clause does not apply. D is incorrect. The Fifteenth Amendment relates to denying citizens the right to vote based on race, color, or previous condition of servitude, which is not applicable here.

A state law provides that a person who has been divorced may not marry again unless he or she is current on all child-support payments. A woman who was refused a marriage license pursuant to this law sued the appropriate state officials. What standard should the court apply in reviewing the constitutionality of this law? A: The state must show that the law is necessary to serve a compelling government interest. B: The state must show that the law is substantially related to an important government interest. C: The woman must show that the law serves no important public purpose. D: The woman must show that the legislature did not have a rational basis for enacting the law.

A is correct. U.S. Supreme Court precedent establishes that an individual's decision to marry is a fundamental right, and therefore, laws that unduly burden a decision to marry trigger strict judicial scrutiny. This strict scrutiny standard obligates the state to prove that the law is necessary to serve a compelling government interest. Here, the law burdens the right to marry by requiring that a person who has been divorced may not remarry unless he or she is current on all child support payments. Because this law burdens the fundamental right to marry, the court will apply the strict scrutiny standard and require the state to prove that the law is necessary to serve a compelling state interest. B is incorrect. This answer states the wrong standard of review. The strict scrutiny standard places a burden on the state that is greater than showing that the law is substantially related to an important government interest. As stated above, it obligates the state to prove that the law is necessary to serve a compelling government interest. C is incorrect. First, it improperly allocates the burden of proof to the woman instead of the state. As explained above, when a law interferes with a fundamental right such as marriage, the court will apply a strict scrutiny standard of review. Second, the standard of review articulated in the question is incorrect. The court will determine whether the law is necessary to serve a compelling government interest, not whether the law serves an important public interest. D is incorrect. This answer improperly allocates the burden of proof to the woman and not the state. This strict scrutiny standard places the burden of proof on the state, not on the woman, and obligates the state to prove that the law is necessary to serve a compelling government interest.

A proposed federal statute would prohibit all types of discrimination against Black persons on the basis of their race in every business transaction executed anywhere in the United States by any person or entity, governmental or private. Is this proposed federal statute likely to be constitutional? A: Yes, because it could reasonably be viewed as an exercise of Congress's authority to enact laws for the general welfare. B: Yes, because it could reasonably be viewed as a means of enforcing the provisions of the Thirteenth Amendment. C: No, because it would regulate purely local transactions that are not in interstate commerce. D: No, because it would invade the powers reserved to the states by the Tenth Amendment.

B is correct. A federal statute prohibiting discrimination on the basis of race in every business transaction could be justified as a means of enforcing the Thirteenth Amendment; it would be rational for Congress to conclude that such legislation serves to eradicate the badges and incidents of slavery. A is incorrect. Congress has the power to enact laws to promote the general welfare in connection with its spending power, and this is not a spending statute. C is incorrect. Congress may, in fact, regulate purely intrastate commercial activities in certain contexts. Furthermore,the business transactions falling within this statute would inherently impact streams of interstate commerce. D is incorrect. The Tenth Amendment does not give states any special powers to regulate business transactions, andCongress has the explicit power to pass this legislation pursuant to the Thirteenth Amendment, as stated above.

A weed often found growing in vacant lots contains a stimulant that can be extracted by boiling the weed in water. Someone drinking the resulting "tea" can remain awake for long periods of time. However, drinking the tea can also cause paralysis and even death. Because the weed is so readily available, there is no commercial market for it. Congress has enacted a statute that makes possession of the tea a misdemeanor. The statute has no other provisions and is not part of a broader federal statutory scheme. Is the statute constitutional? A: No, because Congress can regulate only commercial transactions involving drugs. B: No, because possession of the tea is a noncommercial activity with no link to interstate commerce. C: Yes, because boiling water generally involves the use of stoves and fuels that have moved in interstate commerce. D: Yes, because, in the aggregate, the deaths and cases of paralysis caused by the stimulant have a significant economic impact.

B is correct. Congress does not have the power under the Commerce Clause to regulate a noncommercial activity with no link to interstate commerce. See United States v. Lopez, 514 U.S. 549 (1995). The facts here do not involve a comprehensive regulation of commerce that incidentally covers some intrastate activities, as was the case in Gonzales v. Raich, 545 U.S. 1 (2005). A is incorrect. Congress may use its commerce power to regulate economic activity that is not itself a commercial transaction, such as the manufacturing and production of articles for interstate commerce. Additionally, Congress may incidentally regulate noncommercial activities in order to effectuate regulations of related commercial activities. See Gonzales v. Raich, 545 U.S. 1 (2005). C is incorrect. The statute is outside the scope of congressional commerce powers because, even though stoves andfuels may have previously moved in interstate commerce, they are not channels or instrumentalities of commerce. D is incorrect. The aggregate effects test applies only to regulations of economic activity. United States v. Lopez, 514 U.S. 549 (1995). Here, the statute attempts to prohibit the possession of tea, which is a noncommercial activity with no link to interstate commerce.

Congress recently enacted a statute that provides the US Supreme Court with original jurisdiction over any cause of action against a military officer for official misconduct. The stated intent of the statute is to ensure the prompt disposition of such actions by eliminating the need for appeals. Is the statute constitutional? A: No, because Article II authorizes the President as commander in chief of the armed forces to sanction military misconduct. B: No, because Article III fixes the Supreme Court's original jurisdiction. C: Yes, because Article I vests in Congress the power to establish military tribunals. D: Yes, because Congress has the authority under Article III to define the Supreme Court's original jurisdiction.

B is correct. Congress lacks the constitutional power to add to the Supreme Court's original jurisdiction. See Marbury v. Madison, 5 U.S. 137 (1803). Under Article III Congress may only make exceptions and regulations regarding the Court's appellate jurisdiction, not its original jurisdiction. A is incorrect. Article I of the Constitution, not Article II, gives Congress, not the President, the power to sanction military misconduct. C is incorrect. Congress's Article I power to create military tribunals does not expand its power over the Supreme Court's original jurisdiction. D is incorrect. Congress does not have the authority to define or expand the Supreme Court's original jurisdiction asset by Article III.

The Federal Computer Abuse Act establishes the Federal Computer Abuse Commission, authorizes the Commission to issue licenses for the possession of computers on terms that are consistent with the purposes of the act, and makes the unlicensed possession of a computer a crime. The provisions of the Federal Computer Abuse Act are inseverable. A user applied to the Federal Computer Abuse Commission for a license to possess a computer. The Commission held, and the user participated in, a trial-type proceeding on the user's license application. In that proceeding it was demonstrated that the user repeatedly and intentionally used computers to introduce secret destructive computer programs (computer viruses) into electronic data banks without the consent of their owners. As a result, the Commission denied the user's application for a license. The license denial was based on a Commission rule authorized by the Computer Abuse Act that prohibited the issuance of computer licenses to persons who had engaged in such conduct. Nevertheless, the user retained and continued to use his computer. He was subsequently convicted of the crime of unlicensed possession of a computer. On appeal, he challenges the constitutionality of the licensing provision of the Federal Computer Abuse Act. In this case, the reviewing court would probably hold that act to be A: constitutional, because the Constitution generally authorizes Congress to enact all laws that are necessary and proper to advance the general welfare, and Congress could reasonably believe that possession of computers by people like the user constitutes a threat to the general welfare. B: constitutional, because Congress may use the authority vested in it by the commerce clause to regulate the possession of computers and the provisions of this act do not violate any prohibitory provision of the Constitution. C: unconstitutional, because Congress may not impose a criminal penalty on action that is improper merely because it is inconsistent with an agency rule. D: unconstitutional, because the mere possession of a computer is a wholly local matter that is beyond the regulatory authority of Congress.

B is correct. Congress's broad power under the Commerce Clause allows for the regulation of articles moving in the stream of commerce, which includes regulating the possession of computers. Here, the licensing provision does not violate any other constitutional prohibition, and therefore, is constitutional. A is incorrect. While congressional spending that is deemed necessary and proper must be in service of the general welfare, Congress has no independent right to regulate for the general welfare. C is incorrect. Congress has the power to criminalize certain types of conduct by enacting statutes, which may be prosecuted by executive agencies that have been delegated the authority to do so by Congress. D is incorrect. Nearly all commercial products impact interstate commerce; computers are articles that move in interstate commerce and are not a purely local matter.

In one state, certain advanced diagnostic medical technologies were located only in hospitals, where they provided a major source of revenue. In many other states, such technologies were also available at "diagnostic centers" that were not affiliated with hospitals. A group of physicians announced its plan to immediately open in the state a diagnostic center that would not be affiliated with a hospital. The state hospital association argued to the state legislature that only hospitals could reliably handle advanced diagnostic medical technologies. The legislature then enacted a law prohibiting the operation in the state of diagnostic centers that were not affiliated with hospitals. The group of physicians filed suit challenging the constitutionality of the state law. What action should the court take? A: Uphold the law, because the provision of medical services is traditionally a matter of legitimate local concern that states have unreviewable authority to regulate. B: Uphold the law, because the legislature could rationally believe that diagnostic centers not affiliated with hospitals would be less reliable than hospitals. C: Invalidate the law, because it imposes an undue burden on access to medical services in the state. D: Dismiss the suit without reaching the merits, because the suit is not ripe.

B is correct. Rational basis scrutiny should be applied in this case because there is no impairment of any fundamental right, and the regulation is not based on a suspect or quasi-suspect classification that would trigger heightened judicial scrutiny. Therefore, the law should be upheld because it is rationally related to a legitimate government interest in ensuring that only hospitals, which the legislature has reason to believe are more reliable, should be permitted to handle advanced medical technologies. A is incorrect. Even if the provision of medical services is traditionally a matter of legitimate local concern, states do not have unreviewable authority to regulate it. Constitutional limits still apply to state laws. C is incorrect. Access to medical services is not considered a fundamental right, and therefore, even if there were an"undue burden" placed upon such access, it may still be constitutional because heightened judicial scrutiny is inapplicable. D is incorrect. The lawsuit is ripe because the group of physicians has plans to open a diagnostic center immediately, which means that even though it's a future event, its impact will be immediate.

Congress wishes to enact legislation prohibiting discrimination in the sale or rental of housing on the basis of the affectional preference or sexual preference or sexual orientation of the potential purchaser or renter. Congress wishes this statute to apply to all public and private vendors and lessors of residential property in this country, with a few narrowly drawn exceptions. The most credible argument for congressional authority to enact such a statute would be based upon the A: General Welfare Clause of Article I, Section 8, because the conduct the statute prohibits could reasonably be deemed to be harmful to the national interest. B: Commerce Clause of Article I, Section 8, because, in inseverable aggregates, the sale or rental of almost all housing in this country could reasonably be deemed to have a substantial effect on interstate commerce. C: Enforcement Clause of the Thirteenth Amendment, because that amendment clearly prohibits discrimination against the class of persons protected by this statute. D: Enforcement Clause of the Fourteenth Amendment, because that amendment prohibits all public and private actors from engaging in irrational discrimination.

B is correct. The Commerce Clause offers the most credible support for a statute prohibiting sexual orientation discrimination in public and private sales or rentals because taken together, it could reasonably be found to substantially affect interstate commerce. A is incorrect. The General Welfare Clause empowers Congress to take certain actions in conjunction with its spending powers. This is not a spending statute. C is incorrect. The Thirteenth Amendment is primarily race and ethnicity based, so a statute prohibiting discrimination based on sexual orientation would not fall within this provision. D is incorrect. The Fourteenth Amendment only prohibits state action, not private action, and this statute is directed at private actors as well as state actors.

In an effort to counteract a steep increase in juvenile crime, a state enacted a law terminating the parental rights of any state resident whose child under 16 years of age is convicted of a violent crime in the state. The law directs the state juvenile court to enter a termination order in such a case after the parent has been afforded notice and an opportunity for a hearing at which the only relevant issues are the age of the child and whether the child has been convicted of a violent crime in the state. Is the state law constitutional? A: No, because the law is not narrowly tailored to serve a substantial state interest. B: No, because the law is not necessary to serve a compelling state interest. C: Yes, because a state's police power authorizes it to punish criminal behavior with appropriate sanctions. D: Yes, because the law is rationally related to a legitimate state interest.

B is correct. The Court uses the strict scrutiny standard when a suspect classification or fundamental right is involved. Under this standard, a law will be upheld only if it is necessary to achieve a compelling or overriding government purpose. The custody of one's children has been held to be a fundamental right, and therefore strict scrutiny will apply. Here, there is a state interest in decreasing juvenile crime. However, there are less burdensome ways of decreasing crime. A is incorrect. Although this is the correct conclusion, it applies an incorrect standard. As stated above, strict scrutiny is the applicable test. Under intermediate scrutiny, the law must be substantially related to an important government purpose and is generally used in classifications based on gender or legitimacy. C is incorrect. A state's police power comes from the Tenth Amendment, which gives states the rights and powers not delegated to the United States. States are granted the power to establish and enforce laws protecting the welfare, safety, and health of the public. A state's police power does not authorize it to punish criminal behavior with appropriate sanctions. D is incorrect. This is the standard for rational basis review. The custody of one's children has been held to be a fundamental right, and therefore, strict scrutiny will apply.

Congressional hearings determined that the use of mechanical power hammers is very dangerous to the persons using them and to persons in the vicinity of the persons using them. As a result, Congress enacted a statute prohibiting the use of mechanical power hammers on all construction projects in the United States. Subsequently, a study conducted by a private research firm concluded that nails driven by mechanical power hammers have longer lasting joining power than hand-driven nails. After learning about this study, a city council enacted an amendment to its building safety code requiring the use of mechanical power hammers in the construction of all buildings intended for human habitation. This amendment to the city's building safety code is A: unconstitutional, because it was enacted subsequent to the federal statute. B: unconstitutional, because it conflicts with the provisions of the federal statute. C: constitutional, because the federal statute does not expressly indicate that it supersedes inconsistent state or local laws. D: constitutional, because the long-term safety of human habitation justifies some additional risk to the people engaged in their construction.

B is correct. The Supremacy Clause of the U.S. Constitution provides that a valid federal law controls when a state or local law conflicts with it. The law here is a valid exercise of Congress's commerce powers, and because the local law conflicts with it, the local law is unconstitutional. A is incorrect. The timing of enactment does not matter in a Supremacy Clause analysis. C is incorrect. The Supremacy Clause does not require that a federal law expressly state that it supersedes inconsistent state or local laws. D is incorrect. The city council cannot ignore the Supremacy Clause because it disagrees with the reasoning behind the federal statute due to new information.

A state constitution provides that in every criminal trial "the accused shall have the right to confront all witnesses against him face to face." A defendant was convicted in state court of child abuse based on testimony from a six-yearold child. The child testified while she was seated behind one-way glass, which allowed the defendant to see the child but did not allow the child to see the defendant. The defendant appealed to the state's highest court, claiming that the inability of the child to see the defendant while she testified violated both the United States Constitution and the state constitution. Without addressing the federal constitutional issue, the state's highest court reversed the defendant's conviction and ordered a new trial. The court held that "the constitution of this state is clear, and it requires that while testifying in a criminal trial, a witness must be able to see the defendant." The state petitioned the United States Supreme Court for a writ of certiorari. On which ground should the United States Supreme Court DENY the state's petition? A: A state may not seek appellate review in the United States Supreme Court of the reversal of a criminal conviction by its highest court. B: The decision of the state's highest court was based on an adequate and independent state ground. C: The Sixth Amendment to the United States Constitution does not require that a witness against a criminal defendant be able to see the defendant while the witness testifies. D: The decision of the state's highest court requires a new trial, and therefore it is not a final judgment.

B is correct. The Supreme Court may not review a judgment by the highest court of a state if that judgment is supported entirely by state law and is wholly independent of the interpretation and application of federal law. In this case, although the defendant claimed a violation of the Sixth Amendment of the U.S. Constitution, the state supreme court based its decision entirely on the state constitution without addressing the federal constitutional issue. A is incorrect. The Supreme Court may review a judgment of the highest court of a state if the state court's decision turns on a question arising under federal law. The reason the Supreme Court will deny the petition for certiorari is that the state supreme court based its decision entirely on state law. C is incorrect. The Supreme Court would not reach the merits of the defendant's Sixth Amendment claim. As explained above, the Supreme Court may not review a judgment by the highest court of a state if that judgment is supported entirely by state law and is wholly independent of the interpretation and application of federal law. In this case, although the defendant claimed a violation of the Sixth Amendment to the U.S. Constitution, the state supreme court based its decision entirely on the state constitution without addressing the federal constitutional issue. D is incorrect. Although the Supreme Court may only review final judgments and decrees from the highest state courts, this judgment qualifies because it finally settled the confrontation issue. That issue would not arise again on retrial, and thus, the present petition provided the U.S. Supreme Court its only opportunity to review the confrontation issue. The reason the Supreme Court will deny the petition for certiorari is that the state supreme court based its decision entirely on state law.

A state law provides for an award of damages against anyone who publishes the name of a rape victim. Pursuant to that law, a woman sued a local newspaper in state court after the newspaper identified her as a rape victim. The state trial and appellate courts rejected the claim, holding that the state law was invalid under both the state constitution and the First Amendment of the U.S. Constitution. The state supreme court affirmed, holding specifically: "We think that this well-intentioned law very likely violates the First Amendment of the federal Constitution. We need not, however, decide that issue, because the law assuredly violates our state constitution, which provides even greater protection to the right of the press to report the news." The woman petitioned for review in the U.S. Supreme Court. Is the U.S. Supreme Court likely to review the state supreme court judgment? A: No, because the First Amendment prohibits the imposition of liability for the publication of truthful information. B: No, because the judgment of the state supreme court rests upon an adequate and independent state-law ground. C: Yes, because the supremacy clause does not permit a state to create rights greater than those conferred by the federal Constitution. D: Yes, because the U.S. Supreme Court's appellate jurisdiction extends to cases arising under federal law.

B is correct. The court will not hear a case if it finds adequate and independent non-federal grounds to support the state decision. The grounds must be adequate, meaning they are fully dispositive of the case, so even if the federal grounds are wrongly decided, it will not affect the outcome of the case. The state grounds must also be independent. Meaning, it is not based on any federal law. Here, the state supreme court decided that the law was invalid under the state's constitution. Further, the state's constitution provides even greater protection to the right of the press to report the news. Therefore, the grounds were fully dispositive of the case on adequate and independent state-law ground. A is incorrect. The law is analyzed under the state's constitution. C is incorrect. The Supremacy Clause establishes that federal law takes precedence over state law. It prohibits states from interfering with the federal government's exercise of its constitutional powers. Here, the state law is not interfering with the federal government's exercise of its power, and the Supremacy Clause is not at issue. D is incorrect. As stated above, the Supreme Court would not review the state court judgment. The Supreme Court has appellate jurisdiction through a writ of certiorari or by appeal, not over all cases arising under federal law.

A federal agency solicited bids to build a bridge on an interstate highway. In evaluating the bids, the agency took into account the cost of the bridge and other specified factors. A construction company submitted a bid that was lowest in cost, but the agency chose another bidder that offered more advantages on the other factors. The company that submitted the lowest bid sued the federal agency in federal district court. The only relief that the company sought was to enjoin the issuance of a contract to the prevailing bidder. The company was unable to obtain a temporary restraining order or preliminary injunction. The agency awarded the contract to the bidder it had originally selected. Just as the new bridge was completed and opened, the suit came to trial. How should the court handle this case? A: Award the company the amount of its lost profits. B: Dismiss the suit, because it is now moot. C: Dismiss the suit, because it turns on discretionary decisions of federal officials. D: Hear and determine the case on the merits.

B is correct. The only relief sought by the company was an injunction against awarding the contract to another bidder. During the pretrial period, the contract was awarded to the other bidder and the bridge was completed and opened. Thus, the only relief sought by the company is no longer available and the case is moot. See Alvarez v. Smith, 558 U.S. 87 (2009); De Funis v. Odegaard, 416 U.S. 312 (1974). A is incorrect. The company did not seek damages. The only relief sought by the company was an injunction against awarding the contract to another bidder, and that relief is no longer available. C is incorrect. Discretionary decisions of federal officials are not necessarily immune from review by the courts. D is incorrect. The only relief sought by the company was an injunction against awarding the contract to another bidder, and that relief is no longer available. The case is moot; therefore the court will not consider the merits of the dispute.

"Look-alike drugs" is the term used to describe nonprescription drugs that look like narcotic drugs and are sold on the streets as narcotic drugs. After extensive hearings, Congress concluded that the sale of look-alike drugs was widespread in this country and was creating severe health and law enforcement problems. To combat these problems, Congress enacted a comprehensive statute that regulates the manufacture, distribution, and sale of all nonprescription drugs in the United States. Which of the following sources of constitutional authority can most easily be used to justify the authority of Congress to enact this statute? A: The spending power. B: The Commerce Clause. C: The General Welfare Clause. D: The enforcement powers of the Fourteenth Amendment.

B is correct. The power granted to Congress to regulate commerce has been interpreted so broadly that it encompasses almost any economic act that could conceivably have some kind of impact, even secondarily, on the stream of commerce. Because the sale of these drugs was widespread throughout the country, and the statute targeted the sale, manufacturing, and distribution of the drugs, it is properly authorized by the Commerce Clause. A is incorrect. This is not an action linked with federal spending. These products are not regulated as a condition of receipt of federal funds. C is incorrect. The General Welfare Clause is not an independent source of authority. D is incorrect. The statute seeks to regulate private and public action, and the Fourteenth Amendment applies only to the state action.

A private university is owned and operated by a religious organization. The university is accredited by the department of education of the state in which it is located. This accreditation certifies that the university meets prescribed educational standards. Because it is accredited, the university qualifies for state funding for certain of its operating expenses. Under this funding program, 25 percent of the university's total operating budget comes from state funds. A professor at the university was a part-time columnist for the local newspaper. In one of her published columns, the professor argued that "religion has become a negative force in society." The university subsequently discharged the professor, giving as its sole reason for the dismissal her authorship and publication of this column. The professor sued the university, claiming only that her discharge violated her constitutional right to freedom of speech. The university moved to dismiss the professor's lawsuit on the ground that the U.S. Constitution does not provide the professor with a cause of action in this case. Should the court grant the university's motion to dismiss? A: Yes, because the First and Fourteenth Amendments protect the right of the university to employ only individuals who share and communicate its views. B: Yes, because the action of the university in discharging the professor is not attributable to the state for purposes of the Fourteenth Amendment. C: No, because the accreditation and partial funding of the university by the state are sufficient to justify the conclusion that the state was an active participant in the discharge of the professor. D: No, because the U.S. Constitution provides a cause of action against any state-accredited institution that restricts freedom of speech as a condition of employment.

B is correct. The professor's discharge by the private university did not constitute "state action," which is required to trigger Fourteenth Amendment protections. A is incorrect. The First and Fourteenth Amendments do not grant a university any right to discharge employees who do not share and communicate the views of the university. However, the motion should be granted because the university's discharge of the professor was not "state action." C is incorrect. U.S. Supreme Court precedent establishes that the accreditation, regulation, and partial funding of a university by the state does not render the university's conduct state action, and thus the Fourteenth Amendment does not apply. D is incorrect. The Constitution does not, in fact, contain a provision providing a cause of action against any state-accredited institution that restricts freedom of speech as a condition of employment.

A state law to promote tourism creates an easement across a portion of a private farm for hikers who wish to use a historic trail that crosses the farm. The law allows the farm's owner to charge hikers a fee to cover the costs of this use, but it does not otherwise compensate the owner. Is the law constitutional? A: No, because the fee provision violates the privileges and immunities clause. B: No, because the law takes the owner's property without just compensation. C: Yes, because the fee provides the owner with just compensation. D: Yes, because the law has the public purpose of promoting tourism.

B is correct. The state-created easement provides for the physical occupation of a portion of the owner's farm and thus constitutes a government taking of that portion of land. The Takings Clause of the Fifth Amendment requires that the government provide just compensation to owners for the land it takes. Just compensation is determined by the fair market value of the taken land. Although the state law allows the owner to charge a fee to recover costs from hikers who use the trail on the farm, the fee is not equivalent to the fair market value of an easement across the land. Therefore, the state law is unconstitutional. A is incorrect. The privileges and immunities clause of the Fourteenth Amendment is limited to rights of a special national nature, such as the right to travel. No such rights are at issue here. Thus, the privileges and immunities clause does not apply. C is incorrect. Just compensation is determined by the fair market value of the property taken by the state. Although the state law allows the owner to charge a fee to hikers who use the trail on the farm, the fee is not equivalent to the fair market value of an easement across the land at issue. D is incorrect. The state law is subject to the takings clause, which requires the state to pay just compensation when it takes an individual's property, even if the taking serves a public purpose. Here, the state law does not require the state to pay the farm owner the fair market value of the land taken, which makes the state law unconstitutional.

A city has had a severe traffic problem on its streets. As a result, it enacted an ordinance prohibiting all sales to the public of food or other items by persons selling directly from trucks, cars, or other vehicles located on city streets. The ordinance included an inseverable grandfather provision exempting from its prohibition vendors who, for 20 years or more, have continuously sold food or other items from such vehicles located on the streets of the city. A retail vendor of ice cream products qualifies for this exemption and is the only food vendor that does. A yogurt retailer has a business similar to the ice cream vendor, but the yogurt vendor has been selling to the public directly from trucks located on the streets of the city only for the past ten years. The yogurt vendor filed suit in an appropriate federal district court to enjoin the enforcement of this ordinance on the ground that it denies the yogurt vendor the equal protection of the laws. In this case, the court will probably rule that the ordinance is A: constitutional, because it is narrowly tailored to implement the city's compelling interest in reducing traffic congestion and, therefore, satisfies the strict scrutiny test applicable to such cases. B: constitutional, because its validity is governed by the rational basis test, and the courts consistently defer to economic choices embodied in such legislation if they are even plausibly justifiable. C: unconstitutional, because the nexus between the legitimate purpose of the ordinance and the conduct it prohibits is so tenuous and its provisions are so underinclusive that the ordinance fails to satisfy the substantial relationship test applicable to such cases. D: unconstitutional, because economic benefits or burdens imposed by legislatures on the basis of grandfather provisions have consistently been declared invalid by courts as per se violations of the Equal Protection Clause of the Fourteenth Amendment.

B is correct. This ordinance regulates who may sell food from vehicles on the streets, which implicates neither a fundamental right nor a suspect class. Therefore, the ordinance would be subject to rational basis review, and the court will defer to any economic choice made by the city as long as it is plausibly justifiable. A is incorrect. The ordinance is not subject to strict scrutiny because it does not impact a fundamental right or a suspect class. Moreover, reducing traffic congestion may not be a compelling state interest. C is incorrect. The correct standard is not the "substantial relationship" test as stated here, nor is it necessarily truethat the limitations imposed by the ordinance are so tenuous and under-inclusive such that it would fail this test. D is incorrect. On the contrary, economic benefits or burdens imposed in a way that do not implicate a fundamental right or a suspect class are presumed to be valid under the Equal Protection Clause of the Fourteenth Amendment.

The United States government demonstrated that terrorist attacks involving commercial airliners were perpetrated exclusively by individuals of one particular race. In response, Congress enacted a statute imposing stringent new airport and airline security measures only on individuals of that race seeking to board airplanes in the United States. Which of the following provides the best ground for challenging the constitutionality of this statute? A: The commerce clause of Article I, Section 8. B: The due process clause of the Fifth Amendment. C: The privileges and immunities clause of Article IV. D: The privileges or immunities clause of the Fourteenth Amendment.

B is correct. When the federal government takes action that would violate the Equal Protection Clause of the Fourteenth Amendment had it been conducted by a state government, it amounts to a violation of the Due Process Clause of the Fifth Amendment. The security measure is a presumptive violation of equal protection because it imposes a discriminatory racial classification on one minority group, which triggers strict scrutiny. It would be virtually impossible for the government to prove that the classification is necessary to serve a compelling interest. A is incorrect. The Commerce Clause grants Congress plenary power to regulate the safety of air travel because airlines are instrumentalities of interstate commerce. C is incorrect. The Privileges and Immunities Clause of Article IV is inapplicable because it does not govern the actions of the federal government. Rather, it applies to actions by state governments against citizens of other states. D is incorrect. The Privileges or Immunities Clause of the Fourteenth Amendment has never been applied to actions of the federal government. It only applies to states, prohibiting them from depriving individuals of the privileges or immunities of United States citizenship.

A federal statute imposes an excise tax of $100 on each new computer sold in the United States. It also appropriates the entire proceeds of that tax to a special fund, which is required to be used to purchase licenses for computer software that will be made available for use, free of charge, to any resident of the United States. Is this statute constitutional? A: No, because the federal government may not impose any direct taxes on citizens of the United States. B: No, because the statute takes, without just compensation, the property of persons who hold patents or copyrights on computer software. C: Yes, because it is a reasonable exercise of Congress's powers to tax and spend for the general welfare. D: Yes, because the patent power authorizes Congress to impose reasonable charges on the sale of technology and to spend the proceeds of those charges to advance the use of technology in the United States.

C is correct. Article I, Section 8, Clause 1 of the Constitution gives Congress broad power to tax and to spend for the general welfare. Courts defer to reasonable congressional taxing measures, such as the statute at issue, as well as to expenditures that reasonably further the general welfare. A is incorrect. Article I, Section 9, Clause 4 of the Constitution allows Congress to adopt direct taxes, provided they are in proportion to the national census. Courts defer to reasonable congressional taxing measures, such as the statute at issue, as well as to expenditures that reasonably further the general welfare. B is incorrect. It is true that the Fifth Amendment prohibits the taking of private property for public use without just compensation. But the statute effects no such taking. A tax on the sale of a computer takes no property from those who hold patents or copyrights on computer software. Likewise, the software to be distributed freely under the statute will be purchased, rather than taken, by the government. Courts defer to reasonable congressional taxing measures, such as the statute at issue, as well as to expenditures that reasonably further the general welfare. D is incorrect. Although Article I, Section 8, Clause 8 of the Constitution gives Congress power to provide patent rights to inventors, this Clause does not itself authorize federal taxes and appropriations. Courts defer to reasonable congressional taxing measures, such as the statute at issue, as well as to expenditures that reasonably further the general welfare.

A student contracted for an expensive cable television service for a period of six months solely to view the televised trial of a defendant, who was on trial for murder in a court of a particular state. In the midst of the trial, the judge prohibited any further televising of the defendant's trial because he concluded that the presence of television cameras was disruptive. The student brought an action in federal district court against the judge in the defendant's case asking only for an injunction that would require the judge to resume the televising of the defendant's trial. The student alleged that the judge's order to stop the televising of the defendant's trial deprived him of property--his investment in cable television service--without due process of law. Before the student's case came to trial, the defendant's criminal trial concluded in a conviction and sentencing. There do not appear to be any obvious errors in the proceeding that led to the result in the defendant's case. After the defendant's conviction and sentencing, the opposing party in the student's case moved to dismiss the suit. The most proper disposition of this motion by the federal court would be to A: defer action on the motion until after any appellate proceedings in the defendant's case have concluded, because the defendant might appeal, his conviction might be set aside, he might be tried again, and television cameras might be barred from the new trial. B: defer action on the motion until after the state Supreme Court expresses a view on its proper disposition, because the state law of mootness governs suits in federal court when the federal case is inexorably intertwined with a state proceeding. C: grant the motion, because the subject matter of the controversy between the student and the defendant has ceased to exist and there is no strong likelihood that it will be revived. D: deny the motion, because the student has raised an important constitutional question: whether his investment in cable service, solely to view the defendant's trial, is property protected by the Due Process Clause of the Fourteenth Amendment.

C is correct. Article III limits the exercise of federal court jurisdiction to cases and controversies, and once the injury that gave rise to the case is incapable of remedy, the case becomes moot and no longer justiciable. Here, once the trial was over, the relief the student sought, (televising the trial), was no longer possible, and his case became moot. Moreover, it is unlikely that the controversy will be revived because the case is not on appeal now, and there were no obvious errors in the proceedings. Therefore, the federal court should grant the motion to dismiss. A is incorrect. The chances that the conviction would be set aside are small enough that it is unlikely the controversywould be revived. Moreover, the court might allow cameras for the second trial if one were granted. B is incorrect. There is no state law of mootness. D is incorrect. A federal question cannot be considered without a case or controversy, and a moot issue is not a case or controversy.

A newly enacted federal statute appropriates $100 million in federal funds to support basic research by universities located in the United States. The statute provides that "the ten best universities in the United States" will each receive $10 million. It also provides that "the ten best universities" shall be "determined by a poll of the presidents of all the universities in the nation, to be conducted by the United States Department of Education." In responding to that poll, each university president is required to apply the well-recognized and generally accepted standards of academic quality that are specified in the statute. The provisions of the statute are inseverable. Which of the following statements about this statute is correct? A: The statute is unconstitutional, because the reliance by Congress on a poll of individuals who are not federal officials to determine the recipients of its appropriated funds is an unconstitutional delegation of legislative power. B: The statute is unconstitutional, because the limitation on recipients to the ten best universities is arbitrary and capricious and denies other high quality universities the equal protection of the laws. C: The statute is constitutional, because Congress has plenary authority to determine the objects of its spending and the methods used to achieve them, so long as they may reasonably be deemed to serve the general welfare and do not violate any prohibitory language in the Constitution. D: The validity of the statute is nonjusticiable, because the use by Congress of its spending power necessarily involves political considerations that must be resolved finally by those branches of the government that are closest to the political process.

C is correct. Congress's spending power is plenary, and Congress can choose how it spends its money so long as that choice can reasonably be said to serve the general welfare and not violate any prohibition in the Constitution. A is incorrect. Congress's creation of a polling system to allocate the funds is not an unconstitutional delegation of legislative power. The statute limits the discretion of the university presidents to specific well-recognized and generally accepted standards of academic quality, and so the power granted to the university presidents is very narrow. B is incorrect. Limiting the funding to the ten best universities is a reasonable line to have drawn, given the fact that the funds are limited. D is incorrect. The validity of a statute will nearly always be justiciable and not a political question. This statute is not the type of action and is not on a subject that is clearly committed to the sole discretion of Congress by the Constitution, and ruling on the validity of a statute provides judicially manageable standards.

Congress recently enacted a statute imposing severe criminal penalties on anyone engaged in trading in the stock market who, in the course of that trading, takes "unfair advantage" of other investors who are also trading in the stock market. The statute does not define the term "unfair advantage." There have been no prosecutions under this new statute. The members of an association of law school professors that is dedicated to increasing the clarity of the language used in criminal statutes believe that this statute is unconstitutionally vague. Neither the association nor any of its members is currently engaged in, or intends in the future to engage in, trading in the stock market. The association and its members bring suit against the Attorney General of the United States in a federal district court, seeking an injunction against the enforcement of this statute on the ground that it is unconstitutional. May the federal court determine the merits of this suit? A: Yes, because the suit involves a dispute over the constitutionality of a federal statute. B: Yes, because the plaintiffs seek real relief of a conclusive nature--an injunction against enforcement of this statute. C: No, because the plaintiffs do not have an interest in the invalidation of this statute that is adequate to ensure that the suit presents an Article III controversy. D: No, because a suit for an injunction against enforcement of a criminal statute may not be brought in federal court at any time prior to a bona fide effort to enforce that statute.

C is correct. In order to have standing, a party must have an imminent or actual injury. Here, because the plaintiffs do not trade in the stock market and do not plan to, they have no injury. Accordingly, they lack the standing required to create a case or controversy under Article III. A is incorrect. The presence of a constitutional question cannot make up for a lack of standing. B is incorrect. Although the remedy the plaintiffs seek will redress the injury caused others by this statute, theythemselves have no injury, and lack standing. D is incorrect. A suit to enjoin enforcement of a statute can be brought before a bona fide effort to enforce the statute unless it is clear that the executive plans never to enforce it. An imminent injury is present when someone could be subject to prosecution at any time.

To encourage the growth of its population, a state established a program that awarded $1,000 to the parents of each child born within the state, provided that at the time of the child's birth the mother and father of the newborn were citizens of the United States. A family are aliens who are permanent residents of the United States and have resided in the state for three years. When their first child was born two months ago, they applied for and were denied the $1,000 award by state officials on the sole ground that they are not citizens of the United States. The family filed suit in federal court contending that their exclusion from the award program was unconstitutional. Assume no federal statute addresses this question. In this case, the court should hold that the exclusion of aliens from the state award program is A: constitutional, because the Tenth Amendment reserves to the states plenary authority over the spending of state funds. B: constitutional, because the state has a legitimate interest in encouraging the growth of its population, and a rational legislature could believe that families in which both parents are United States citizens are more likely to stay in the state and contribute to its future prosperity than those in which one or both of the parents are aliens. C: unconstitutional, because strict scrutiny governs judicial review of such state classifications based on alienage, and the state cannot demonstrate that this classification is necessary to advance a compelling state interest. D: unconstitutional, because state classifications based on alienage are impermissible unless explicitly authorized by an act of Congress.

C is correct. State laws on alienage are subject to strict scrutiny unless they discriminate against alien participation in state government (in which case only a rational basis review would be required). Since the state cannot demonstrate that this classification is necessary to advance a compelling state interest, it will not pass a strict scrutiny review. A is incorrect. Any power to spend state funds is necessarily limited by the Fourteenth Amendment. B is incorrect. Rational basis is the incorrect level of review; since this is state law, alienage is a suspect classification. D is incorrect. State statutes can make a classification based on alienage as long as the classification is necessary to advance a compelling state interest.

Insurance is provided in a particular state only by private companies. Although the state insurance commissioner inspects insurance companies for solvency, the state does not regulate their rates or policies. An insurance company charges higher rates for burglary insurance to residents of one part of a county in the state than to residents of another section of the same county because of the different crime rates in those areas. The plaintiff is a resident of the county who was charged the higher rate by the insurance company because of the location of her residence. The plaintiff sues the insurance company, alleging that the differential in insurance rates unconstitutionally denies her the equal protection of the law. Will the plaintiff's suit succeed? A: Yes, because the higher crime rate in the plaintiff's neighborhood demonstrates that the county police are not giving persons who reside there the equal protection of the laws. B: Yes, because the insurance rate differential is inherently discriminatory. C: No, because the constitutional guarantee of equal protection of the law is not applicable to the actions of these insurance companies. D: No, because there is a rational basis for the differential in insurance rates.

C is correct. The Constitution provides for equal protection of the law, which means that it protects individuals from actions by the state (the Fourteenth Amendment) or the federal government (the Fifth Amendment). Equal protection only restricts private action in extremely specific circumstances, none of which are present in this fact pattern. A is incorrect. This choice assumes that the Constitution prohibits differential treatment by private companies.Moreover, the plaintiff is suing the insurance company, not the police, for a violation of equal protection. The equal protection violation here would be caused by the police. B is incorrect. Generally, private discrimination does not violate the Constitution. D is incorrect. The Constitution only restricts private action in extremely specific circumstances, none of which are present in this situation.

A state law provides some funding for public schools on a per-student basis from general state revenues, which primarily come from the state income and sales taxes. The law also provides that all other public monies used to support public schools in the state come from locally levied real estate taxes. This results in a large disparity in per-student funding among the state's many public school districts because some districts have higher property values per student than other districts. Public school students who claim to be disadvantaged by this school funding law have challenged the law solely on the ground that it violates the Fourteenth Amendment's equal protection clause. Which of the following best states the burden of persuasion in this action? A: The state must demonstrate that the law is necessary to vindicate a compelling state interest. B: The state must demonstrate that the law is rationally related to a legitimate state interest. C: The students must demonstrate that the law is not rationally related to any legitimate state interest. D: The students must demonstrate that the law is not substantially related to an important state interest.

C is correct. The Court will apply one of three standards when examining governmental action involving classifications of persons. If a suspect classification or fundamental right is involved, strict scrutiny will apply. If a quasi-suspect classification is involved, the government must prove that the action is substantially related to an important government interest. If any other classification is involved, the action will be upheld unless the action is not rationally related to a legitimate government interest. The burden is on the plaintiff under rational basis review. Here, public school students are not part of a suspect or quasi-suspect class, and rational basis applies. A is incorrect. This is the standard for strict scrutiny. As stated above, rational basis applies. B is incorrect. The burden of persuasion is the responsibility of persuading the trier of fact that the fact at issue is true.The party that bears the burden of persuasion must satisfy that burden in order to prevent a decision against it. The state would not have the burden of persuasion in this action because the public school students are the ones bringing the action. D is incorrect. This is the standard for intermediate scrutiny. Under intermediate scrutiny, the law must be substantially related to an important government purpose. Under intermediate scrutiny, it is unclear who has the burden of proof, but in most cases, it appears to be the government. In this case, the standard of rational basis applies.

A state statute permits a person's name to appear on the general election ballot as a candidate for statewide public office if the person pays a $100 filing fee and provides proof from the State Elections Board that he or she was nominated in the immediately preceding primary election by one of the state's two major political parties. It also permits the name of an independent candidate or a candidate of a smaller party to appear on the general election ballot if that person pays a filing fee of $1,000, and submits petitions signed by at least 3% of the voters who actually cast ballots for the office of governor in the last state election. The state maintains that these filing requirements are necessary to limit the size of the election ballot, to eliminate frivolous candidacies, and to help finance the high cost of elections. Historically, very few of the state's voters who are members of racial minority groups have been members of either of the two major political parties. Recently, a new political party has been formed by some of these voters. Which of the following constitutional provisions would be most helpful to the new political party as a basis for attacking the constitutionality of this statute? A: The First Amendment. B: The Thirteenth Amendment. C: The Fourteenth Amendment. D: The Fifteenth Amendment.

C is correct. The Fourteenth Amendment prohibits a state from limiting access to the ballot to members of particular political parties unless the prohibition is necessary to further a compelling interest. This is therefore the provision most helpful to the new political party in its efforts to attack the constitutionality of the statute. A is incorrect. The First Amendment does contain a right of association that is injured by this statute, but the FirstAmendment only applies to the states by operation of the Fourteenth Amendment. B is incorrect. The statute classifies on the basis of political party rather than race, and the party members are fromseveral racial minority groups, not just Black people, and therefore the classification may not be a badge and incident of slavery that the Thirteenth Amendment works to eradicate. D is incorrect. The classification does not limit the right of voters on the basis of race.

State A spends several million dollars a year on an oyster conservation program. As part of that program, the state limits, by statute, oyster fishing in its coastal waters to persons who have state oyster permits. In order to promote conservation, it issues only a limited number of oyster permits each year. The permits are effective for only one year from the date of their issuance and are awarded on the basis of lottery, in which there is no differentiation between resident and nonresident applicants. However, each nonresident who obtains a permit is charged an annual permit fee that is $5 more than the fee charged residents. A large fishing company operates from a port in another state and is incorporated in that other state. Each of the boats of the fishing company has a federal shipping license that permits it "to engage in all aspects of the coastal trade, to fish and to carry cargo from place to place along the coast, and to engage in other lawful activities along the coast of the United States." These shipping licenses are authorized by federal statute. Assume no other federal statutes or administrative rules apply. Although it had previously held a State A oyster permit, the fishing company did not obtain a permit in that state's lottery this year. Which of the following is the strongest argument that can be made in support of a continued right of the fishing company to fish for oysters this year in the coastal waters of State A? A: Because the state law provides higher permit charges for nonresidents, it is an undue burden on interstate commerce. B: Because the state law provides higher permit charges for nonresidents, it denies the fishing company the privileges and immunities of state citizenship. C: Because it holds a federal shipping license, the fishing company has a right to fish for oysters in state waters despite the state law. D: Because the fishing company previously held a State A oyster permit and the state knows that the company is engaged in a continuing business operation, the refusal to grant the fishing company a permit this year is a taking of its property without due process of law.

C is correct. The Supremacy Clause ensures that any conflict between a valid federal law and a state law, as is the case here, the federal law will prevail and thus, the company will be able to continue fishing based on its federal license. A is incorrect. The fishing company was denied a state permit, so a higher rate for permit charges is irrelevant to the fishing company which was denied the permit entirely. B is incorrect. The Article IV Privileges and Immunities Clause is inapplicable to corporations, and a higher charge is not relevant to the fishing company, which was denied the license entirely. D is incorrect. The permit is a property right which only lasts one year, and so there was no lasting property that the state could have taken.

A state statute requires each insurance company that offers burglary insurance policies in the state to charge a uniform rate for such insurance to all of its customers residing within the same county in that state. So long as it complies with this requirement, a company is free to charge whatever rate the market will bear for its burglary insurance policies. An insurance company located in the state files suit in federal district court against appropriate state officials to challenge this statute on constitutional grounds. The insurance company wishes to charge customers residing within the same county in the state rates for burglary insurance policies that will vary because they would be based on the specific nature of the customer's business, on its precise location, and on its past claims. In this suit, the court should A: hold the statute unconstitutional, because the statute deprives the insurance company of its liberty or property without due process of law. B: hold the statute unconstitutional, because the statute imposes an undue burden on interstate commerce. C: hold the statute constitutional, because the statute is a reasonable exercise of the state's police power. D: abstain from ruling on the merits of this case until the state courts have had an opportunity to pass on the constitutionality of this state statute.

C is correct. The court should hold the statute constitutional here because it is a reasonable exercise of the state's police power. The state has a legitimate interest in regulating the sale of insurance. And, because the limit the state puts on insurance companies is uniform and very minimal (that it must charge a uniform rate to all customers within the same county but may charge whatever rate that the market will bear), that limit is reasonable and not otherwise unconstitutional. A is incorrect. The state has not substantially interfered with the use of any insurance company property that isprotected by the Due Process Clause. B is incorrect. The treatment of all insurance companies is uniform, and the statute imposes no undue burden oninterstate commerce because the same requirements apply to policies offered by all companies. D is incorrect. There is no reason to abstain. This is not a situation in which the meaning of the state statute is unsettled, or that involves an area that is so fully regulated by the state that the federal court, in the interest of good relations would abstain.

Congress recently enacted a statute creating a program that made federal loans available to family farmers who had been unable to obtain loans from private lenders. Congress appropriated a fixed sum of money to fund loans made pursuant to the program and gave a designated federal agency discretion to decide which applicants were to receive the loans. Two weeks after the program was established, a family farmer applied to the agency for a loan. Agency officials promptly reviewed her application and summarily denied it. The farmer has sued the agency in federal district court, claiming only that the denial of her application without the opportunity for a hearing violated the due process clause of the Fifth Amendment. The farmer claims that she could have proved at such a hearing that without the federal loan it would be necessary for her to sell her farm. Should the court uphold the agency's decision? A: No, because due process requires federal agencies to provide a hearing before making any factual determination that adversely affects an identified individual on the basis of his or her particular circumstances. B: No, because the denial of a loan may deprive the farmer of an established liberty interest to pursue her chosen occupation. C: Yes, because the applicable statute gives the farmer no legitimate claim of entitlement to receive a loan. D: Yes, because the spending clause of Article I, Section 8, gives Congress plenary power to control the distribution of appropriated funds in any manner it wishes.

C is correct. The court should uphold the agency's decision because the Due Process Clause does not require the government to provide the farmer with an opportunity for an administrative hearing on her loan application. The farmer had no legitimate claim of entitlement to a loan because the statute gave the agency discretion to decide which applicants were to receive the loans. The agency's denial of the farmer's application, therefore, did not deprive her of a property or liberty interest protected by the Due Process Clause. A is incorrect. Due process obligates agencies to provide an individual with an opportunity for a hearing only when the agency makes an adjudicatory decision that deprives the individual of a property or liberty interest that is protected. As explained above, the agency's decision should be upheld because the Due Process Clause does not require the government to provide the farmer an opportunity for an administrative hearing on her loan application because she had no legitimate claim of entitlement to a loan. B is incorrect. The farmer's decision to pursue her chosen occupation does not qualify as a liberty interest protected by the Due Process Clause. D is incorrect. This answer reaches the correct answer with the wrong reasoning. Although Congress has broad authority to control the distribution of appropriated funds, that authority is subject to many constitutional limitations on the legislative power, including the Due Process Clause of the Fifth Amendment.

Radon is a harmful gas found in the soil of certain regions of the United States. A state statute requires occupants of residences with basements susceptible to the intrusion of radon to have their residences tested for the presence of radon and to take specified remedial steps if the test indicates the presence of radon above specified levels. The statute also provides that the testing for radon may be done only by testers licensed by a state agency. According to the statute, a firm may be licensed to test for radon only if it meets specified rigorous standards relating to the accuracy of its testing. These standards may easily be achieved with current technology; but the technology required to meet them is 50% more expensive than the technology required to measure radon accumulations in a slightly less accurate manner. The United States Environmental Protection Agency (EPA) does not license radon testers. However, a federal statute authorizes the EPA to advise on the accuracy of various methods of radon testing and to provide to the general public a list of testers that use methods it believes to be reasonably accurate. A recently established state firm uses a testing method that the EPA has stated is reasonably accurate. The firm is also included by the EPA on a list of testers using methods of testing it believes to be reasonably accurate. The firm applies for a state radon testing license, but its application is denied because the firm cannot demonstrate that the method of testing for radon it uses is sufficiently accurate to meet the rigorous state statutory standards. The firm sues appropriate state officials in federal court claiming that the state may not constitutionally exclude the firm from performing the required radon tests in the state. In this suit, the court will probably rule in favor of A: the firm, because the Full Faith and Credit Clause of the Constitution requires the state to respect and give effect to the action of the EPA in including the firm on its list of testers that use reasonably accurate methods. B: the firm, because the Supremacy Clause of the Constitution requires the state to respect and give effect to the action of the EPA in including the firm on its list of testers that use reasonably accurate methods. C: the state, because the federal statute and the action of the EPA in including the firm on its list of testers that use reasonably accurate methods are not inconsistent with the more rigorous state licensing requirement, and that requirement is reasonably related to a legitimate public interest. D: the state, because radon exposure is limited to basement areas, which, by their very nature, cannot move in interstate commerce.

C is correct. The federal statute is very general and does not regulate radon testers to the extent of licensing them. Therefore, the general federal statute does not conflict with the more rigorous state licensing standards, and those licensing standards are not superseded by the federal law under the Supremacy Clause. Furthermore, the state's licensing requirement is constitutional because it is reasonably related to a legitimate public interest. A is incorrect. The Full Faith and Credit Clause does not require a state to accept an action by the federal government that would not comport with its laws as long as those laws are not superseded by the Supremacy Clause or otherwise constitutionally invalid. B is incorrect. The federal statute does not conflict with the state statute. It provides less regulation of the area, rather than more. D is incorrect. The testers provide a service that is part of the stream of commerce. Additionally, the effects of radon, the process of testing for it, and remedying its effects all have commercial components. Thus, regulating radon levels is within Congress's commerce power.

With the advice and consent of the Senate, the President entered into a self-executing treaty with a foreign country. The treaty provided that citizens of both nations were required to pay whatever tort damages were awarded against them by a court of either nation. A man and a woman who were U.S. citizens and residents of the same state were traveling separately in the foreign country when their cars collided. The foreign court awarded the woman a judgment for $500,000 in damages for her injuries from the accident. The woman filed suit against the man in federal district court in their home state to enforce the judgment. The man filed a motion to dismiss for lack of jurisdiction. Should the court grant the motion to dismiss? A: Yes, because the citizenship of the parties is not diverse. B: Yes, because the traffic accident was a noncommercial transaction outside interstate commerce. C: No, because the case falls within the federal question jurisdiction of the court. D: No, because the treaty power is plenary and not subject to judicial review.

C is correct. The motion should be denied because, under Article III, federal courts have federal question jurisdiction over cases arising under a treaty of the United States, as is the case here. A is incorrect. Although the court does lack diversity jurisdiction, it nevertheless has federal question jurisdiction because the case is arising under a treaty of the U.S., as provided by Article III of the Constitution. B is incorrect. Whether the traffic accident was a non-commercial transaction is irrelevant to whether the federal district court has jurisdiction over this case, which is proper, given that it involves a treaty of the U.S. under Article III. D is incorrect. Even though the President has broad power to execute treaties, that is not relevant to the issue presented in this question. The issue is whether the federal district court has the power to exercise jurisdiction over this case, which it does under federal question jurisdiction.

A city owned and operated an electric utility that supplied electricity to the city's businesses and residences. Citing the need for efficiency, the city implemented a plan to replace old electric meters with wireless meters in businesses and residences. The city's plan did not include advance notice to property owners or a method to obtain consent from property owners. A homeowner who opposed the installation of the wireless meters was denied his request for a hearing on the city's plan. The homeowner then blocked the city's access to his house, obstructing the city's effort to replace the old meter. The city responded by threatening to arrest and prosecute the homeowner for disorderly conduct. The homeowner then sued the city in federal district court, challenging the meter-replacement plan on due process grounds and seeking to enjoin its enforcement. The city moved to dismiss. While the city's motion was pending, the city council passed an emergency ordinance requiring both prior notice and property owner consent before the removal or replacement of any electric meter. Should the court grant the city's motion? A: No, because the city's action is capable of repetition yet evading review. B: No, because the city's voluntary cessation of the challenged conduct does not moot the case. C: Yes, because the case is moot. D: Yes, because the case is not ripe.

C is correct. The only relief sought by the homeowner is no longer available, making the case moot. See Allready LLC v. Nike, Inc., 568 U.S. 85 (2013). A is incorrect. "Capable of repetition yet evading review" applies only in situations where litigation is unlikely to be completed because of the short-term nature of an issue. That is not the case here. B is incorrect. The voluntary cessation exception does not apply when it is unreasonable to expect that the unlawful conduct will recur. Here, the emergency ordinance's explicit requirements of prior notice and owner consent make it unreasonable to expect that the city will attempt to change the homeowner's meter. In addition, the only relief sought by the homeowner was an injunction against the previous ordinance, which is no longer in effect. D is incorrect. Ripeness is determined when a suit is filed. Here, the case was ripe, given the city's attempts to install the meter and its threats of prosecution.

A nightclub owner applied for a required zoning permit to open a nude-dancing nightclub in the theater district of a city. An organization of influential city residents began an intensive lobbying effort to persuade the city council to deny the owner a permit to operate any type of nude- dancing facility at any time or in any place in the city. The owner has sued the city in an appropriate federal court, seeking an injunction that would prohibit the city council from considering the organization's views, on the ground that if the organization is successful in its lobbying efforts, the owner's First and Fourteenth Amendment rights would be violated. The city has moved to dismiss the action. Should the court dismiss the owner's action? A: No, because nude dancing is symbolic speech and is therefore protected by the First and Fourteenth Amendments. B: No, because the organization does not seek a reasonable time, place, and manner regulation of nude dancing, but instead seeks a total ban on the owner's opening any type of nude-dancing facility at any time or in any place in the city. C: Yes, because the action is not ripe. D: Yes, because the First and Fourteenth Amendments do not protect obscenity, and nude dancing is obscene.

C is correct. The owner's action is based on an uncertain and contingent future event - if the council hears the organization's views, the organization would then succeed in its lobbying efforts and subsequently violate his constitutional rights. Based on these facts, the matter has not sufficiently matured for the court to rule on it. A is incorrect. The court should dismiss the owner's action because the action is not ripe. The fact that nude dancing is protected by the First Amendment is irrelevant to the analysis. B is incorrect. As stated above, the action is not ripe. Therefore, an analysis of time, place, and manner is unnecessary. D is incorrect. This is a misstatement of the law. Nude dancing does have First Amendment protections. The case should nevertheless be dismissed because it is not yet ripe.

A city's police officers shot and killed a plaintiff's friend as he attempted to escape arrest for an armed robbery he had committed. The plaintiff brought suit in federal district court against the police department and the city police officers involved, seeking only a judgment declaring unconstitutional the state statute under which the police acted. That newly enacted statute authorized the police to use deadly force when necessary to apprehend a person who has committed a felony. In his suit, the plaintiff alleged that the police would not have killed his friend if the use of deadly force had not been authorized by the statute. The federal district court should A: decide the case on its merits, because it raises a substantial federal question. B: dismiss the action, because it involves a nonjusticiable political question. C: dismiss the action, because it does not present a case or controversy. D: dismiss the action, because the Eleventh Amendment prohibits federal courts from deciding cases of this type.

C is correct. The plaintiff does not have standing required to establish his stake in the case or controversy, which is necessary for proper federal action. The plaintiff has not suffered an injury; it was a third party (his friend) who was shot and killed. Moreover, any injury the plaintiff does have from his friend dying cannot be redressed by a finding of unconstitutionality. A is incorrect. Even if the statute's constitutionality raises a substantial federal question, a person with standing mustbe the one to challenge it for there to be a case or controversy, as required by Article III. B is incorrect. The statute's constitutionality does not present a political question - a question best resolved by thefunctions of the political process. D is incorrect. The Eleventh Amendment only prohibits suits against the state, and this suit was against the city police department and individual officers. Cities are not equivalent to a state for purposes of the Eleventh Amendment.

A baseball fan has a fierce temper and an extremely loud voice. Attending a baseball game in which a number of calls went against the home team, the fan repeatedly stood up, brandished his fist, and angrily shouted, "Kill the umpires." The fourth time he engaged in this conduct, many other spectators followed the fan in rising from their seats, brandishing fists, and shouting, "Kill the umpires." The home team lost the game. Although no violence ensued, spectators crowded menacingly around the umpires after the game. As a result, the umpires were able to leave the field and stadium only with the help of a massive police escort. For his conduct, the fan was charged with inciting to riot and was convicted in a jury trial in state court. He appealed. The state supreme court reversed his conviction. In its opinion, the court discussed in detail decisions of the United States Supreme Court dealing with the First Amendment Free Speech Clause as incorporated into the Fourteenth Amendment. At the end of that discussion, however, the court stated that it "need not resolve how, on the basis of these cases," the United States Supreme Court would decide the fan's case. Instead, the court stated, "this court has always given the free-speech guarantee of the state's constitution the broadest possible interpretation. As a result, we hold that in this case, where no riot or other violence actually occurred, the state constitution does not permit this conviction for incitement to riot to stand." The United States Supreme Court grants a writ of certiorari to review this decision of the state supreme court. In this case, the United States Supreme Court should A: affirm the state supreme court's decision, because the fan's ballpark shout is commonplace hyperbole that cannot, consistently with the First and Fourteenth Amendments, be punished. B: remand the case to the state supreme court with directions that it resolve the First and Fourteenth Amendment free-speech issue that it discussed in such detail. C: dismiss the writ as improvidently granted, because the state supreme court's decision rests on an independent and adequate state law ground. D: reverse the decision of the state supreme court, because incitement to violent action is not speech protected by the First and Fourteenth Amendments.

C is correct. The state court's decision rested on an independent and adequate state-law ground because it explicitly held that it would not resolve how the Supreme Court would have adjudicated the matter based on federal law, but rather, it would give the state constitutional provision "the broadest possible interpretation" and it "does not permit this conviction for incitement to riot to stand." (emphasis added). The Court should, therefore, dismiss the writ as improvidently granted. A is incorrect. It is possible that the speech would not be protected under the federal Constitution if it was "fighting words" or if it posed a "clear and present danger." Nevertheless, because an independent and adequate state-law ground existed, the Court should dismiss the writ. B is incorrect. As stated above, the state court does not need to resolve the federal issue because its opinion relied upon independent and adequate state grounds. D is incorrect. Because an independent and adequate state-law ground existed, the writ should be dismissed and the analysis should end there. However, the speech might not have been considered an incitement to violent action given that there was some delay between the speech and the menacing of the umpires and no violence broke out.

A doctor who was licensed to practice medicine in a particular state was convicted in state court of improperly distributing specified drugs by writing prescriptions for fictitious persons. Under state law, such an abuse of the prescription-writing privilege requires revocation of a doctor's license. After it received an official notification of the doctor's conviction, the state medical board revoked the doctor's license without affording the doctor any opportunity for a hearing. The doctor has sued the board in state court to set aside the revocation, alleging deprivation of property without due process of law because the board did not provide an opportunity for a trial-type hearing before revoking the license. The doctor does not deny the conviction or the factual basis for it. Which of the following is the strongest argument in support of the state medical board? A: A doctor's license to practice is a privilege, not a right, and therefore is not property within the meaning of the due process clause of the Fourteenth Amendment. B: Due process requires a balancing of interests, and the state's interest in preventing drug abuse outweighs the doctor's interest in the particular procedure followed in the disciplinary proceeding. C: The adjudicative facts necessary to revoke the doctor's license were determined in the criminal trial, and therefore due process does not require any further trial-type hearing. D: The licensing board was required to summarily revoke the doctor's license because Article IV, Section 1, of the Constitution requires the licensing board to give full faith and credit to the doctor's criminal conviction.

C is correct. The strongest argument in support of the board's revocation of the license without a hearing is the fact that the doctor already received extensive procedural due process protections through his trial as a criminal defendant, which would render an additional hearing unnecessary. A is incorrect. The Supreme Court has recognized that a doctor's license is considered "property" within the meaning of the Due Process Clause, and is not merely a privilege without such constitutional protections. B is incorrect. Even though courts do balance interests in determining procedural due process claims, this is not the strongest argument here. The doctor had a criminal trial where he had the opportunity to present his defense and adjudicate all the relevant facts to his revocation. D is incorrect. The Full Faith and Credit Clause states that full faith and credit should be given in each state to the public acts, records, and judicial proceedings of every other state. It does not govern state medical boards.

A state statute declares that after five years of continuous service in their positions, all state employees, including faculty members at the state university, are entitled to retain their positions during "good behavior." The statute also contains a number of procedural provisions. Any state employee who is dismissed after that five-year period must be given reasons for the dismissal before it takes effect. In addition, such an employee must, upon request, be granted a post-dismissal hearing before an administrative board to seek reinstatement and back pay. The statute precludes any other hearing or opportunity to respond to the charges. That post-dismissal hearing must occur within six months after the dismissal takes effect. The burden of proof at such a hearing is on the state, and the board may uphold the dismissal only if it is supported by a preponderance of the evidence. An employee who is dissatisfied with a decision of the board after a hearing may appeal its decision to the state courts. The provisions of this statute are inseverable. A teacher who had been employed continuously for seven years as a faculty member at the state university was dismissed. A week before the dismissal took effect, she was informed that she was being dismissed because of a charge that she accepted a bribe from a student in return for raising the student's final grade in her course. At that time she requested an immediate hearing to contest the propriety of her dismissal. Three months after her dismissal, she was granted a hearing before the state administrative board. The board upheld her dismissal, finding that the charge against her was supported by a preponderance of the evidence presented at the hearing. The faculty member did not appeal the decision of the state administrative board to the state courts. Instead, she sought a declaratory judgment in federal district court to the effect that the state statute prescribing the procedures for her dismissal is unconstitutional. In this case, the federal district court should A: dismiss the suit, because a claim that a state statute is unconstitutional is not ripe for adjudication by a federal court until all judicial remedies in state courts provided for by state law have been exhausted. B: hold the statute unconstitutional because the Due Process Clause of the Fourteenth Amendment requires a state to demonstrate beyond a reasonable doubt the facts constituting good cause for termination of a state employee. C: hold the statute unconstitutional, because a state may not ordinarily deprive an employee of a property interest in a job without giving the employee a pre-termination opportunity to respond to the charges against that employee. D: hold the statute constitutional, because the Due Process Clause of the Fourteenth Amendment entitles state employees who have a right to their jobs during good behavior only to a statement of reasons for their dismissal and an opportunity for a post-dismissal hearing.

C is correct. The teacher's procedural due process rights have been violated because of the deprivation of her property interest in continued public employment. She had the right to receive notice of her termination and a pretermination hearing with an opportunity to respond. However, she was not given a hearing until after her termination was effective, and therefore, the statute is unconstitutional. A is incorrect. The court can properly hear this case, which is ripe for adjudication. It falls within the jurisdiction of the federal judiciary because it involves a constitutional claim. It is not necessary that the teacher first exhaust all state avenues before bringing a claim in federal court. Moreover, the case is ripe because an actual injury has occurred the teacher has, in fact, lost her job. B is incorrect. Procedural due process does not require the state to prove good cause beyond a reasonable doubt for the employee's termination. Rather, the court will conduct a balancing test and weigh the strength of the plaintiff's interest in receiving a particular procedural safeguard against the government's interest in avoiding extra burdens from having to give that safeguard. D is incorrect. State employment is considered an important enough property right to require a pre-dismissal hearing, and typically a post-dismissal hearing is only constitutionally acceptable when a pre-termination hearing would be considered highly impracticable, which is not the case here.

Public schools in a state are financed, in large part, by revenue derived from real estate taxes imposed by each school district on the taxable real property located in that district. Public schools also receive other revenue from private gifts, federal grants, student fees, and local sales taxes. For many years, the state has distributed additional funds, which come from the state treasury, to local school districts in order to equalize the funds available on a perstudent basis for each public school district. These additional funds are distributed on the basis of a state statutory formula that considers only the number of students in each public school district and the real estate tax revenue raised by that district. The formula does not consider other revenue received by a school district from different sources. The school boards of two school districts, together with parents and schoolchildren in those districts, bring suit in federal court to enjoin the state from allocating the additional funds from the state treasury to this formula. They allege that the failure of the state, in allocating this additional money, to take into account a school district's sources of revenue other than revenue derived from taxes levied on real estate located there violates the Fourteenth Amendment. The complaint does not allege that the allocation of the additional state funds based on the current statutory formula has resulted in a failure to provide minimally adequate education to any child. Which of the following best describes the appropriate standard by which the court should review the constitutionality of the state statutory funding formula? A: Because classifications based on wealth are inherently suspect, the state must demonstrate that the statutory formula is necessary to vindicate a compelling state interest. B: Because the statutory funding formula burdens the fundamental right to education, the state must demonstrate that the formula is necessary to vindicate a compelling state interest. C: Because no fundamental right or suspect classification is implicated in this case, the plaintiffs must demonstrate that the funding allocation formula bears no rational relationship to any legitimate state interest. D: Because the funding formula inevitably leads to disparities among the school districts in their levels of total funding, the plaintiffs must only demonstrate that the funding formula is not substantially related to the furtherance of an important state interest.

C is correct. There is no fundamental right to education, and wealth is not a suspect class. Because there is no fundamental right or suspect class at issue, the court will apply rational basis review, which would require the plaintiffs to demonstrate that the funding allocation formula is not rationally related to any conceivable legitimate state interest. A is incorrect. Wealth is not a suspect class. B is incorrect. Education is not a fundamental right. D is incorrect. The regulation only needs to be rationally related to a legitimate state interest.

A state legislature recently enacted a statute forbidding public utilities regulated by the state's public service commission to increase their rates more than once every two years. A power company, a public utility regulated by that commission, has just obtained approval of the commission for a general rate increase. The power company has routinely filed for a rate increase every ten to 14 months during the last 20 years. Because of uncertainties about future fuel prices, the power company cannot ascertain with any certainty the date when it will need a further rate increase; but it thinks it may need such an increase sometime within the next 18 months. The power company files an action in the federal district court in the state requesting a declaratory judgment that this new state statute forbidding public utility rate increases more often than once every two years is unconstitutional. Assume no federal statute is relevant. In this case, the court should A: hold the statute unconstitutional, because such a moratorium on rate increases deprives utilities of their property without due process of law. B: hold the statute constitutional, because the judgment of a legislature on a matter involving economic regulation is entitled to great deference. C: dismiss the complaint, because this action is not ripe for decision. D: dismiss the complaint, because controversies over state-regulated utility rates are outside of the jurisdiction conferred on federal courts by Article III of the Constitution.

C is correct. There is no injury yet to the power company. It has not yet been denied a rate increase, and it does not even know whether it will need to seek a rate increase before the statute will allow it to. Therefore, there is not yet an actual or imminent injury, and the claim is not yet ripe for decision. A is incorrect. There is no vested property right that the company can claim has been injured. B is incorrect. There has been no injury that would allow a court to really determine whether this legislation is rationally related to a legitimate interest. D is incorrect. Article III provides no exemption for state-regulated utility rates. And, in fact, if the parties presented a case or controversy, the constitutionality of the rate regulation would raise a federal question.

Congress enacted a statute prohibiting discrimination in the rental of residential property anywhere in the United States on the basis of sexual orientation or preference by any person or entity, public or private. Which of the following provisions provides the strongest basis for Congress's authority to enact this statute A: The enforcement clause of the Fourteenth Amendment. B: The privileges and immunities clause of Article IV. C: The commerce clause of Article I, Section 8. D: The general welfare clause of Article I, Section 8.

C is correct. This statute regulating residential property rental terms, although it may individually occur intrastate, relates to economic activity that, in the aggregate, has a substantial effect on interstate commerce. The strongest basis for the statute's constitutionality is, therefore, the plenary congressional power to regulate interstate commerce. A is incorrect. The Fourteenth Amendment's Enforcement Clause applies to congressional regulation of state action. The Enforcement Clause would therefore not support the application of the federal statute to private individuals and entities. B is incorrect. The Privileges and Immunities Clause of Article IV prevents states from discriminating against non-residents and is not a basis for federal congressional action. D is incorrect. The General Welfare Clause applies only to taxing and spending legislation passed by Congress.

A state law requires any lawn mower sold in the state to meet a specified minimum level of fuel efficiency. A new federal statute requires all power equipment, including lawn mowers, to be labeled with energy efficiency stickers to permit purchasers to make informed choices when buying such equipment. The statute does not expressly preempt state law. Assume that no other federal statute or administrative regulation addresses the energy efficiency of power equipment. Which of the following is the best argument the state can make for the continued validity of its law? A: Congress cannot preempt state laws requiring a specified minimum level of fuel efficiency for lawn mowers, because the use of such equipment is a wholly local event and, therefore, is beyond the regulatory authority vested in Congress by the commerce clause. B: The law is unaffected by the federal statute, because Congress did not expressly prohibit state laws requiring power equipment to meet specified levels of fuel efficiency. C: The purpose of the law is consistent with the purpose of the federal statute, enforcement of the law would not interfere with the full execution of the statute, and there is no evidence that Congress intended to preclude the states from enacting supplemental laws on this subject. D: There is a very strong presumption that a specific state law on a subject normally within the state's police power prevails over a more general federal statute, because the Tenth Amendment reserves to the states primary authority over matters affecting public health, welfare, and safety.

C is correct. When states and the federal government pass legislation on the same subject matter, the Supremacy Clause applies. The facts state that the statute does not expressly preempt state law. However, even if federal law does not expressly prohibit state action, state laws will be held impliedly preempted if they conflict with federal requirements. Therefore, the best argument that the state can make would be that the purpose of the law is consistent with the federal statute, and enforcement would not interfere with the full execution of the statute. A is incorrect. The Commerce Clause has been held to be interpreted expansively. Therefore, this is not the best argument the state can make. B is incorrect. Even if Congress did not expressly prohibit the state law, state law will be held impliedly preempted if they conflict with federal requirements, prevent achievement of federal objectives, or if Congress has preempted the entire field. D is incorrect. While this is a true statement, it does not apply here. The state police power refers to the capacity of the states to regulate behavior and enforce order within their territory. Here, the state law is regulating fuel efficiency so this does not apply.

A city ordinance requires a taxicab operator's license to operate a taxicab in that city. The ordinance states that the sole criteria for the issuance of such a license are driving ability and knowledge of the geography of the city. An applicant is tested by the city for these qualifications with a detailed questionnaire, written and oral examinations, and a practical behind-the-wheel demonstration. The ordinance does not limit the number of licenses that may be issued. It does, however, allow any citizen to file an objection to the issuance of a particular license, but only on the ground that an applicant does not possess the required qualifications. City licensing officials are also authorized by the ordinance to determine, in their discretion, whether to hold an evidentiary hearing on an objection before issuing a license. A woman applies for a taxicab operator's license and is found to be fully qualified after completing the usual licensing process. Her name is then posted as a prospective licensee, subject only to the objection process. A licensed taxicab driver files an objection to the issuance of such a license to the woman solely on the ground that the grant of a license to the woman would impair the value of the licensed driver's existing license. The licensed driver demands a hearing before a license is issued to the woman so that he may have an opportunity to prove his claim. City licensing officials refuse to hold such a hearing, and they issue a license to the woman. The licensed driver petitions for review of this action by city officials in an appropriate court, alleging that the Constitution requires city licensing officials to grant his request for a hearing before issuing a license to the woman. In this case, the court should rule for A: the licensed driver, because the Due Process Clause of the Fourteenth Amendment requires all persons whose property may be adversely affected by governmental action to be given an opportunity for a hearing before such action occurs. B: the licensed driver, because the determination of whether to hold a hearing may not constitutionally be left to the discretion of the same officials whose action is being challenged. C: city officials, because the licensed driver had the benefit of the licensing ordinance and, therefore may not now question actions taken under it. D: city officials, because the licensing ordinance does not give the licensed driver any property interest in being free of competition from additional licensees.

D is correct. A person may not be deprived of liberty or property without procedural due process. The city ordinance is the source of the property right alleged here, but the scope of that property right is limited. Taxicab licenses do not include a right to be free of competition from additional licensees. Furthermore, the ordinance does not limit the number of licenses that may be issued and provides for a hearing only when the ground alleged is a lack of qualifications. Therefore, the grant of a license to the woman does not deprive the licensed driver of any property right that would require procedures to protect it. Moreover, there is no statutory right to a hearing because the only ground for a hearing is a lack of qualification, and the decision of whether to grant a pre-termination hearing is up to the discretion of the city licensing officials. A is incorrect. There is no property right at stake here, which means that it does not trigger any requirement of adequate procedural "process." B is incorrect. The right to a hearing is only implicated when a property right is threatened. The discretion of the officials over the hearing is not a problem. C is incorrect. A person who has the benefit of a process that grants some kind of property right can challenge the deprivation of that right.

A mineral is added to bodies of fresh water to prevent the spread of certain freshwater parasites. The presence of those parasites threatens the health of organisms living in rivers and streams throughout the country and imperils the freshwater commercial fishing industry. The mineral is currently mined only in one particular state. In order to raise needed revenue, Congress recently enacted a statute providing for the imposition of a $100 tax on each ton of the mineral that is mined in the United States. Because it will raise the cost of the mineral, this tax is likely to reduce the amount of the mineral that is added to freshwater rivers and streams and, therefore, is likely to have an adverse effect on the interstate freshwater commercial fishing industry. The mineral producers in the state have filed a lawsuit in federal court challenging this tax solely on constitutional grounds. Is this tax constitutional? A: No, because only producers in the state will pay the tax and, therefore, it is not uniform among the states and denies mineral producers the equal protection of the laws. B: No, because it is likely to have an adverse effect on the freshwater commercial fishing industry and Congress has a responsibility under the clause to protect, foster, and advance such interstate industries. C: Yes, because the tax is a necessary and proper means of exercising federal authority over the navigable waters of the United States. D: Yes, because the power of Congress to impose taxes is plenary, this tax does not contain any provisions extraneous to tax needs or purposes, and it is not barred by any prohibitory language in the Constitution.

D is correct. Congress's power to tax is plenary, and a tax measure will be upheld if it bears any reasonable relationship to the goal of revenue generation. This statute does not contain provisions extraneous to tax purposes, and it does not violate any other section of the Constitution. A is incorrect. Mineral producers are not suspect or quasi-suspect as a class, which means under a rational basis review, the tax law would have to be arbitrary or irrational, which it is not. The tax is also uniform in its application, it just happens that the mineral is only mined in a single state, where anyone can come to mine it. B is incorrect. Congress does have the power to regulate interstate commerce including the fishing industry, it is not required to choose one interest over another. Congress need only determine, based on competing interests, the best course of action for rationally carrying out its constitutionally-enumerated powers. C is incorrect. The tax is justified under Congress's taxing power, not as a subsidiary to Congress's power to control the navigable waters of the United States.

The legislature of a particular state enacted a statute requiring that all law enforcement officers in that state be citizens of the United States. An alien, lawfully admitted to permanent residency five years before the enactment of this statute, sought employment as a forensic pathologist in the state coroner's office. He was denied such a job solely because he was not a citizen. The alien thereupon brought suit in federal district court against appropriate state officials seeking to invalidate this citizenship requirement on federal constitutional grounds. The strongest ground upon which to attack this citizenship requirement is that it A: constitutes an ex post facto law as to previously admitted aliens. B: deprives an alien of a fundamental right to employment without the due process of law guaranteed by the Fourteenth Amendment. C: denies an alien a right to employment in violation of the Privileges or Immunities Clause of the Fourteenth Amendment. D: denies an alien the equal protection of the laws guaranteed by the Fourteenth Amendment.

D is correct. The Equal Protection Clause requires that classifications based on alienage be narrowly tailored to promote a compelling state interest. However, strict scrutiny for alienage classifications does not apply where the discrimination against aliens relates to functions that go to the heart of the representative government. This means that individuals who hold state-elected or important non-elected executive positions, and those who are government officers that participate directly in the formulation, execution, or review of broad public policy may be required to be citizens. If the law discriminates against an alien participating in state government, the standard is rational basis review. It has been held that the government may discriminate against aliens with respect to the following positions: state troopers; public school teachers; jurors; and deputy probation officers. In this case, it is unclear which standard the position of forensic pathologist would be evaluated against. However, because this law is treating a group of people differently, the best challenge would be under equal protection. A is incorrect. The law does not criminalize an act after it was committed, nor does it impose a burden on a vested right; therefore it is not an ex post facto law. B is incorrect. Employment is not a fundamental right. C is incorrect. The Privileges or Immunities Clause prohibits discrimination by states against United States citizens, and an alien is not a United States citizen.

Twenty percent of the residents of Green City are members of minority racial groups. These residents are evenly distributed among the many different residential areas of the city. The five city council members of Green City are elected from five single-member electoral districts that are nearly equally populated. No candidate has ever been elected to the city council who was a member of a minority racial group. A group of citizens who are members of minority racial groups file suit in federal district court seeking a declaratory judgment that the single-member districts in Green City are unconstitutional. They claim that the single-member districting system diminishes the ability of voters who are members of minority racial groups to affect the outcome of the city elections. They seek an order from the court forcing the city to adopt an at-large election system in which the five candidates with the greatest vote totals would be elected to the city council. No state or federal statutes are applicable to the resolution of this suit. Which of the following constitutional provisions provides the most obvious basis of plaintiff's claim in this suit? A: The Thirteenth Amendment. B: The Due Process Clause of the Fourteenth Amendment. C: The Privileges or Immunities Clause of the Fourteenth Amendment. D: The Fifteenth Amendment.

D is correct. The Fifteenth Amendment provides that the right to vote shall not be abridged on the basis of race or color. Because this is a challenge by minority racial groups to the way voting is conducted, the Fifteenth Amendment is the most obvious basis for their claim. A is incorrect. The Thirteenth Amendment abolishes slavery but does not focus solely on the right to vote. TheFifteenth Amendment provides a better basis for their claim because it focuses on voting rights. B is incorrect. The Due Process Clause of the Fourteenth Amendment gives Congress the power to remedy violations of individual rights by the government, but only as those rights have been defined by the courts. While the Fourteenth Amendment is designed to promote racial equality, it does not protect the right to vote explicitly. C is incorrect. The Privileges or Immunities Clause of the Fourteenth Amendment offers a congressional remedy for individual rights violations by the government, but again, only as defined by the courts. As explained above, the Fourteenth Amendment does not explicitly protect the right to vote.

A city owns and operates a large public auditorium. It leases the auditorium to any group that wishes to use it for a meeting, lecture, concert, or contest. Each user must post a damage deposit and pay rent, which is calculated only for the actual time the building is used by the lessee. Reservations are made on a first-come, first-served basis. A private organization that permits only males to serve in its highest offices rented the auditorium for its national convention. The organization planned to install its new officers at that convention. It broadly publicized the event, inviting members of the general public to attend the installation ceremony at the city auditorium. No statute or administrative rule prohibits the organization from restricting its highest offices to men. An appropriate plaintiff sues the private organization seeking to enjoin it from using the city auditorium for the installation of its new officers. The sole claim of the plaintiff is that the use of this auditorium by the organization for the installation ceremony is unconstitutional because the organization disqualifies women from serving in its highest offices. Will the plaintiff prevail? A: Yes, because the Fourteenth Amendment prohibits such an organization from discriminating against women in any of its activities to which it has invited members of the general public. B: Yes, because the organization's use of the city auditorium for this purpose subjects its conduct to the provisions of the Fourteenth Amendment. C: No, because the freedom of association protected by the Fourteenth Amendment prohibits the city from interfering in any way with the organization's use of city facilities. D: No, because the organization is not a state actor and, therefore, its activities are not subject to the provisions of the Fourteenth Amendment.

D is correct. The Fourteenth Amendment prohibits state action but not private action. Therefore, the prohibition of women from serving as officers in this organization is not unconstitutional. A is incorrect. The Fourteenth Amendment does not reach this private actor, and also because the private actor does not become a state actor merely by inviting the public to an activity. B is incorrect. The use of the auditorium does not discriminate on the basis of gender; women can come to the installation of new officers, and the use of the auditorium does not make the organization a state actor. C is incorrect. The city could regulate who would be allowed to use its facilities, although some regulations might have to pass strict scrutiny.

The United States Department of the Interior granted a concessionaire the food and drink concession stand in a federal park located in a particular state. The concessionaire operated his concession stand out of the federally owned facilities in the park. The federal statute authorizing the Interior Department to grant such concession stands provided that the grantees would pay only a nominal rental fee for use of these federal facilities because of the great benefit their concession stands would provide to the people of the United States. The legislature of the state enacted a statute imposing an occupancy tax on the occupants of real estate within that state that is not subject to state real estate taxes. The statute was intended to equalize the state tax burden on such occupants with that of people occupying real estate that is subject to state real estate taxes. Pursuant to that statute, the state Department of Revenue attempted to collect the state occupancy tax from the concessionaire because the federal facilities occupied by the concessionaire were not subject to state real estate taxes. The concessionaire sued to invalidate the state occupancy tax as applied to him. The strongest ground upon which the concessionaire could challenge the occupancy tax is that it violates the A: Commerce Clause by unduly burdening the interstate tourist trade. B: Privileges or Immunities Clause of the Fourteenth Amendment by interfering with the fundamental right to do business on federal property. C: Equal Protection Clause of the Fourteenth Amendment because the tax treats him less favorably than federal concessionaires in other states who do not have to pay such occupancy. D: Supremacy Clause of Article VI and the federal statute authorizing such concession stands.

D is correct. The Supremacy Clause of Article VI provides that validly-enacted federal laws supersede conflicting state and local laws. The federal concession statute is valid under the property power, which allows Congress to make rules and regulations regarding property belonging to the United States. Thus, the federal law will supersede the state law and insulate the activity taking place on federal property from state taxation. A is incorrect. A tax on an activity does not by itself burden the interstate tourist trade as long as that tax applies togoods and services equally no matter where produced. Here there is no such distinction. B is incorrect. There is no fundamental right to do business on federal property, and therefore, it cannot be a right ofnational citizenship protected under the Fourteenth Amendment Privileges or Immunities Clause. C is incorrect. The state tax does not classify on the grounds of any suspect class.

A federal statute extends federal minimum wage requirements to all dry cleaning stores. The statute contains express findings that, when combined, the wages received by dry cleaning workers have a substantial impact on the national economy and on the flow of goods and services in interstate commerce. These findings are supported by information presented to Congress during committee hearings on the legislation. A small dry cleaning store operates exclusively within a community in the center of a geographically large state. It has no customers from outside the state. It employs three workers, each of whom is paid less than the federal minimum wage. Must this dry cleaning store comply with the statute imposing the federal minimum wage requirements on all dry cleaning stores? A: No, because the store does no business in interstate commerce. B: No, because the wages of the store's three workers do not have a substantial impact on interstate commerce. C: Yes, because the commerce clause vests Congress with plenary legislative authority over labor relations. D: Yes, because the wages paid by dry cleaning stores have a substantial impact on interstate commerce.

D is correct. The Supreme Court has held that Congress has the power to regulate any activity, local or interstate, that has a substantial effect upon interstate commerce. As the facts state, the wages received by dry cleaning workers have a substantial impact on the national economy and on the flow of goods and services in interstate commerce. A is incorrect. Although the store does no business in interstate commerce, the facts indicate that wages made by dry cleaning employees have a substantial effect upon interstate commerce. The Supreme Court has held that Congress may regulate any activity that in itself or in a combination with other activities has a substantial economic effect upon interstate commerce. B is incorrect. The wages of the store's three workers may not have a substantial impact on interstate commerce, but the wages combined by all dry cleaning workers have a substantial impact on the national economy. Therefore, Congress may regulate federal minimum wage requirements. C is incorrect. The Commerce Clause does not vest Congress with plenary legislative authority over labor relations.As stated above, the Commerce Clause regulates activities that have a substantial effect on interstate commerce.

In response to the need for additional toxic waste landfills in a state, the state's legislature enacted a law authorizing a state agency to establish five new state-owned and state-operated toxic waste landfills. The law provided that the agency would decide the locations and sizes of the landfills after an investigation of all potential sites and a determination that the particular sites chosen would not endanger public health and would be consistent with the public welfare. A community in the state was scheduled for inspection by the agency as a potential toxic waste landfill site. Because the community's residents obtained most of their drinking water from an aquifer that ran under the entire community, a citizens' group, made up of residents of that community, sued the appropriate officials of the agency in federal court. The group sought a declaratory judgment that selecting their community as the site of a toxic waste landfill would be unconstitutional and an injunction preventing the agency from doing so. The agency officials moved to dismiss. Which of the following is the most appropriate basis for the court to dismiss this suit? A: The case presents a nonjusticiable political question. B: The interest of the state in obtaining suitable sites for toxic waste landfills is sufficiently compelling to justify the selection of the community as a location for such a facility. C: The Eleventh Amendment bars suits of this kind in the federal courts. D: The case is not ripe for a decision on the merits.

D is correct. The case arguably is not ripe for adjudication because the agency's inspection does not itself pose any risk of harm to residents of the community. The residents face a risk of harm only if the agency selects their community as a site for a landfill, but on these facts it is unclear whether or when the community would be selected. A is incorrect. The political question doctrine insulates from judicial review certain constitutional questions that theConstitution has committed either to the legislative branch or to the executive branch of the federal government. No such question is presented on these facts, which concern actions by a state government. B is incorrect. There are no facts to suggest that strict judicial scrutiny of the state's site-selection decision is warranted. Therefore, the state need not show that the selection of the community is necessary to serve a compelling interest. A more appropriate basis for a court dismissing the suit would be that the case is not ripe for adjudication. C is incorrect. The Eleventh Amendment does not bar the suit because it was brought against state officers, not the state itself, and because it seeks only prospective declaratory and injunctive relief, not compensatory monetary relief.

In order to foster an environment conducive to learning, a school board enacted a dress code that prohibited all public high school students from wearing in school shorts cut above the knee. Because female students at the school considered it unfashionable to wear shorts cut at or below the knee, they no longer wore shorts to school. On the other hand, male students at the school regularly wore shorts cut at or below the knee because they considered such shorts to be fashionable. Female students sued to challenge the constitutionality of the dress code on the ground that it denied them the equal protection of the laws. Should the court uphold the dress code? A: No, because the dress code is not necessary to further a compelling state interest. B: No, because the dress code is not substantially related to an important state interest. C: Yes, because the dress code is narrowly tailored to further an important state interest. D: Yes, because the dress code is rationally related to a legitimate state interest.

D is correct. The court should uphold the dress code because it is rationally related to the state's legitimate interest in fostering a proper educational environment. The dress code should not trigger heightened judicial scrutiny because there are no facts to suggest that the purpose of the code is to discriminate against female students. A is incorrect. As discussed above, the court is applying rational basis review to the dress code. This answer choice isincorrect because it applies to a strict level of scrutiny, which is not appropriate for the interest involved in the dress code. B is incorrect. As explained above, the dress code should not trigger heightened judicial scrutiny because there are nofacts to suggest that the purpose of the code is to discriminate against female students. C is incorrect. While the court should uphold the dress code, this answer choice incorrectly assumes the dress code would trigger heightened scrutiny.

A federal statute requires the National Bureau of Standards to establish minimum quality standards for all beer sold in the United States. The statute also provides that public hearings must precede adoption of the standards, and that once they are adopted, the standards will be subject to judicial review. While the proposed standards have not yet been announced, several Bureau officials have publicly expressed opinions indicating a belief that pasteurized beer is safer than unpasteurized beer. However, these officials have not stated whether they intend to include a pasteurization requirement in the standards. A brewery that produces unpasteurized beer is concerned that, after the appropriate proceedings, the Bureau may adopt quality standards that will prohibit the sale of unpasteurized beer. The brewery has sued in federal district court to enjoin the Bureau from adopting standards that would prohibit the sale of unpasteurized beer. How should the district court proceed with the suit? A: Determine whether the Bureau could reasonably believe that pasteurization is the safest process by which to brew beer and, if so, refuse to issue the injunction against the Bureau. B: Determine whether the process used by the brewery is as safe as pasteurization and, if so, issue the injunction against the Bureau. C: Refuse to adjudicate the merits of the suit at this time and stay the action until the Bureau has actually issued beer quality standards. D: Refuse to adjudicate the merits of the suit and dismiss it, because it does not involve a justiciable case or controversy.

D is correct. The federal courts lack the power to entertain a suit that is not ripe for adjudication because such a suit does not present a "case" or "controversy" within the meaning of Article III, Section 2, Clause 1 of the Constitution. The district court should dismiss the suit because the Bureau has yet to announce the beer-quality standards, and therefore the case is not ripe. The court may not maintain jurisdiction over the suit by issuing a stay because it lacks the constitutional authority to retain control over the suit. A is incorrect. Because the case does not satisfy the requirements for ripeness under Article III, Section 2, Clause 1 of the Constitution, it should be dismissed without hearing the merits. B is incorrect. As stated above, the case is not yet ripe because there have been no beer-quality standards announced, and therefore, it should not proceed. C is incorrect. The district court should not stay the action, it should dismiss it because the suit does not present a case or controversy within the meaning of the U.S. Constitution.

An interstate bus company operates in a five-state area. A federal statute authorizes the Interstate Commerce Commission (ICC) to permit interstate carriers to discontinue entirely any unprofitable route. The interstate bus company applied to the ICC for permission to drop a very unprofitable route through the sparsely populated Shaley Mountains. The ICC granted that permission even though the interstate bus company provided the only public transportation into the region. A man is the owner of a mountain resort in the Shaley Mountains, whose customers usually arrived on vehicles operated by the interstate bus company. After exhausting all available federal administrative remedies, the man filed suit against the interstate bus company in the trial court of the state in which the Shaley Mountains are located to enjoin the discontinuance by the interstate bus company of its service to that area. The man alleged that the discontinuance of service by the interstate bus company would violate a statute of that state prohibiting common carriers of persons from abandoning service to communities having no alternate form of public transportation. The state court should A: dismiss the action, because the man lacks standing to sue. B: direct the removal of the case to federal court, because this suit involves a substantial federal question. C: hear the case on its merits and decide for the man because, on these facts, a federal agency is interfering with essential state functions. D: hear the case on its merits and decide for the interstate bus company, because a valid federal law preempts the state statute on which the man relies.

D is correct. The man is suing under a state statute that forbids carriers from discontinuing routes to areas with no alternative means of public transit. This directly conflicts with a federal statute that authorizes transit companies to discontinue routes that are not profitable. Thus, the federal statute will preempt the state statute under the Supremacy Clause. A is incorrect. The man does have standing. He has an injury to his business that was caused by the federal action, and prohibiting the discontinuance would remedy his injury. B is incorrect. States have concurrent jurisdiction to decide federal questions since state courts are courts of general jurisdiction. C is incorrect. Regulating interstate common carriers is not an essential state function; rather, it is a quintessential federal function.

A local high school has had a very high rate of pregnancy among its students. In order to help students who keep their babies to complete high school, the local high school has established an infant day-care center for children of its students and also offers classes in childcare. Because the childcare classes are always overcrowded, the school limits admission to those classes solely to the high school students who are mothers of babies in the infant day-care center. A student at the high school has legal custody of his infant son. The school provides care for his son in its infant daycare center but will not allow him to enroll in the childcare classes. He brings suit against the school challenging, on constitutional grounds, his exclusion from the childcare classes. Which of the following best states the burden of persuasion in this case? A: The student must demonstrate that the admission requirement is not rationally related to a legitimate governmental interest. B: The student must demonstrate that the admission requirement is not as narrowly drawn as possible to achieve a substantial governmental interest. C: The school must demonstrate that the admission policy is the least restrictive means by which to achieve a compelling governmental interest. D: The school must demonstrate that the admission policy is substantially related to an important governmental interest.

D is correct. The school's policy classifies on the basis of gender, a quasi-suspect class, and thus is subject to intermediate scrutiny. This means that the government, and not the individual, bears the burden of persuasion in showing that the policy is substantially related to an important governmental interest. A is incorrect. Rational basis review is not the applicable standard because the policy regulates based on gender, a quasi-suspect class subject to intermediate scrutiny. B is incorrect. This is the incorrect standard to apply here, where the school bears the burden and only intermediate scrutiny applies. C is incorrect. Strict scrutiny does not apply to gender classifications, as stated above.

Congress enacted a statute authorizing the denial of all federal funding to public school districts in which a specified percentage of the students enrolled in the public schools fail to pass a national achievement test. According to the terms of the federal statute, the first national achievement test was scheduled for administration five years from the effective date of the statute. After reviewing then-current levels of public school student performance, the officials of a state became concerned that several of its public school districts would lose their federal funding after the administration of the first national achievement test. Then-current levels of private school student performance were substantially higher. In order to improve the chances of those school districts retaining their federal funding, the state recently enacted a law that requires all children of elementary and secondary school age to attend the schools operated by their respective local public school districts. The law is to take effect at the beginning of the next school year. Parents of children enrolled in private schools within the state have filed suit to challenge the constitutionality of this state law. Should the court uphold the law? A: Yes, because it is rationally related to a legitimate state interest. B: Yes, because it is necessary to further a compelling state interest. C: No, because it is not rationally related to a legitimate state interest. D: No, because it is not necessary to further a compelling state interest.

D is correct. The state's law requiring kids to attend public schools defies the Court's holding in Pierce that such laws are invalid. Moreover, the law infringes upon parents' fundamental right to raise their children, which triggers strict scrutiny, a level of review that cannot be satisfied here. A is incorrect. Even if this law would satisfy rational basis review, this is not the proper standard because it infringesupon a fundamental right, thus triggering strict scrutiny, as explained above. B is incorrect. Even though this response states the proper standard of review, it misstates the court's likely finding. Asexplained above, the law is not necessary to further a compelling state interest. C is incorrect. The law is rationally related to a legitimate state interest. However, this is the incorrect standard ofreview. The court would apply strict scrutiny, and the law would not pass this higher standard of review.

A statute in a particular state provided state monetary grants to private dance, theater, and opera groups located in that state. The statute required recipients of such grants to use the granted monies for the acquisition, construction, and maintenance of appropriate facilities for the public performance of their performing arts. The last section of the statute conditioned the award of each such grant on the recipient's agreement to refrain from all kinds of political lobbying calculated to secure additional tax support for the performing arts. The strongest constitutional basis for an attack upon the validity of the last section of the statute would be based upon the A: Commerce Clause. B: Obligation of Contracts Clause. C: Fifth Amendment. D: First and Fourteenth Amendments.

D is correct. The strongest constitutional basis for an attack on the provision is that by conditioning the award of a grant on the recipient's agreement to refrain from all kinds of political lobbying designed to secure additional tax support, the state was prohibiting a particular kind of speech or political activity, in violation of the First and Fourteenth Amendments. State actions that restrict speech or similar political activity are subject to strict scrutiny and rarely pass. A is incorrect. The state action does not impair the flow of interstate commerce since it does not restrict a commercialactivity so much as a political activity. B is incorrect. No existing contract is being impaired by the statute. C is incorrect. The Fifth Amendment applies only to the federal government, not the states.

A city zoning ordinance requires that anyone who proposes to operate a group home obtain a special use permit from the city zoning board. The zoning ordinance defines a group home as a residence in which four or more unrelated adults reside. An individual applied for a special use permit to operate a group home for convicts during their transition from serving prison sentences to their release on parole. Although the proposed group home met all of the requirements for the special use permit, the zoning board denied the individual's application because of the nature of the proposed use. The individual sued the zoning board seeking declaratory and injunctive relief on constitutional grounds. Which of the following best states the appropriate burden of persuasion in this action? A: Because housing is a fundamental right, the zoning board must demonstrate that denial of the permit is necessary to serve a compelling state interest. B: Because the zoning board's action has the effect of discriminating against a quasi-suspect class in regard to a basic subsistence right, the zoning board must demonstrate that the denial of the permit is substantially related to an important state interest. C: Because the zoning board's action invidiously discriminates against a suspect class, the zoning board must demonstrate that denial of the permit is necessary to serve a compelling state interest. D: Because the zoning board's action is in the nature of an economic or social welfare regulation, the individual seeking the permit must demonstrate that the denial of the permit is not rationally related to a legitimate state interest.

D is correct. The zoning board's denial of the permit did not discriminate against a suspect or a quasi-suspect class, nor did it unduly burden the exercise of a fundamental right. The denial therefore triggers rational basis scrutiny. A is incorrect. The Supreme Court has not held that housing is a fundamental right. Therefore, the zoning board's denial of the permit does not trigger strict scrutiny on that basis. B is incorrect. The Court has not held that convicts constitute a quasi-suspect class. Consequently, the zoning board's denial of the permit does not trigger intermediate scrutiny on that basis. C is incorrect. As stated above, the Court has not held that convicts constitute a suspect class. The zoning board's denial of the permit thus does not trigger strict scrutiny on that basis.

The National AIDS Prevention and Control Act is a new, comprehensive federal statute that was enacted to deal with the public health crisis caused by HIV/AIDS. Congress and the President were concerned that inconsistent lower court rulings with respect to the constitutionality, interpretation, and application of the statute might adversely affect or delay its enforcement and, thereby, jeopardize the public health. As a result, they included a provision in the statute providing that all legal challenges concerning those matters may be initiated only by filing suit directly in the United States Supreme Court. The provision authorizing direct review of the constitutionality, interpretation, or application of this statute only in the United States Supreme Court is A: constitutional, because it is authorized by the Article I power of Congress to enact all laws that are "necessary and proper" to implement the general welfare. B: constitutional, because Article III provides that the jurisdiction of the United States Supreme Court is subject to such exceptions and such regulations as Congress shall make. C: unconstitutional, because it denies persons who wish to challenge this statute the equal protection of the laws by requiring them to file suit in a court different from that in which persons who wish to challenge other statutes may file. D: unconstitutional, because it is inconsistent with the specification in Article III of the original jurisdiction of the United States Supreme Court.

D is correct. This provision is inconsistent with Article III, which states that the Court has original jurisdiction in cases involving ambassadors, ministers and consuls, and cases in which a state is a party. None of these scenarios applies here. A is incorrect. While the statute itself is valid under the Necessary and Proper Clause, the judicial review provision can only be authorized under Article III. B is incorrect. Congress only has the power to make exceptions and regulations governing the Supreme Court's appellate jurisdiction. C is incorrect. The judicial power of the United States is exercised by any Article III court, including the SupremeCourt. There is no differential treatment as long as an Article III court has jurisdiction at some point in the process.

A city passed an ordinance requiring individuals to obtain a license in order to care for children under the age of 12 for pay. City officials who promoted the ordinance said that the licensing process would ensure that child-care workers were adequately regulated for the health and safety of the city's children. To receive a license, the ordinance requires an individual to complete 10 hours of instruction in child care, undergo a background check, and pay a $100 fee. The ordinance affects women disproportionately to men, because female child-care workers far outnumber male child-care workers in the city. Is the ordinance constitutional? A: No, because it has a disparate impact on women without a showing that the ordinance is necessary to serve a compelling government interest. B: No, because it infringes on the freedom of contract without a compelling government interest. C: Yes, because any burden it imposes is clearly outweighed by an important government objective. D: Yes, because it is rationally related to a legitimate government objective.

D is correct. Without any evidence that the city passed the ordinance in order to disproportionately affect women, it does not amount to sex discrimination and must only satisfy mere rationality review, which it does, given its legitimate interest in protecting the health and safety of children through licensing regulation. A is incorrect. The city does not have to prove that the disparate impact on women is necessary to serve a compelling state interest, which is the strict scrutiny level of review. Even if the disparate impact on women had been intentional, the correct level of judicial review would be intermediate scrutiny. However, as explained above, the ordinance's disparate impact on women, absent a discriminatory intent, need only satisfy mere rationality review. B is incorrect. Even if the ordinance was found to infringe upon the freedom of contract, strict judicial scrutiny is inappropriate because the freedom of contract is not a fundamental right. C is incorrect. The appropriate standard of judicial review is not whether the burdens imposed by the ordinance outweigh the benefits, but whether it is rationally related to a legitimate government objective.


Conjuntos de estudio relacionados

Pediatric Succes - Chapter 6 - Cardiovascular Disorders

View Set

Brain and Behavior Chapter 6 Review

View Set

Surgical Tech- Chapter 9: Surgical Pharmacology and Anesthesia

View Set

Lab Quiz 6 Respiratory Physiology

View Set

BMGT 1327 - PRINCIPLES OF MANAGEMENT - CH 16 ASSIGNMENT

View Set

MKTG 3333 Digital Marketing Ch. 4

View Set